שחזור ילדים ועידת ברצי

You might also like

Download as pdf or txt
Download as pdf or txt
You are on page 1of 78

Only new questions- 10.6.

2022
1. Low Ca, high PO4, high PTH can be seen in: ‫)י‬473( 486 ‫בספר הפולני עמ‬
a. Hypoparathyroidism
b. Hyperparathyroidism
c. Pseudo-hyperparathyroidism ‫דניאל‬
d. No correct answer

2. What is the most common cause of dyspnea in children below the age of 5?
a. Respiratory infection
b. Obstruction by foreign body

3. What is the definition of obesity in children? ‫)י‬233( 246 ‫ספר פולני עמ‬
a. BMI > 95th
b. BMI > 98th- ‫ליאל‬
c. Body mass > 98
d. 2+3

4. Approach to the child with aspiration of foreign body with no chest movement and respiration
a. Start CPR ‫דניאל‬
b. Abdominal compressions
c. Blow 5 times then PCR
d. Blow 3 times then PCR
e. Only observation for deterioration

5. Child with CF and diabetes, what is the best treatment?


a. Metformin
b. Insulin ‫דניאל‬
c. DPP4 inhibitor
d. SLGT2 inhibitor

6. Which of the following combinations are symptoms of secondary Addison?


a. Hyperpigmentation of lips
b. Hypopigmentation of nipples
c. Metabolic acidosis ‫דניאל‬

7. Body surface percent of child after burn of one upper arm + anterior thigh
a. 7% - ‫דניאל‬- ‫ למרות שלא מסתדר‬,‫ לפי המרצה במבחן‬-‫ליאל‬
b. 6.75%- ‫במידה והאמה נחשבת ליד עליונה (ללא הצד השני) וירך לפי ילד בן שנה‬
c. 6%

8. Cushing disease symptoms


a. Hypertension
b. Hirsutism
c. Hypercalcemia? ‫דניאל מה זה‬
d. All

9. The statement about strep. Pneumonia


a. The leading cause of pneumonia in all ages
b. If penicillin resistant treat with vanco -the gerbil
c. If penicillin resistant treat with macrolides
.‫ שירן‬-macrolides ‫ אם יש רגישות לפניצילין אז ניתן‬,‫ לעמידות‬3 ‫ דור‬cephalosporins ‫ הטיפול צריך להיות עם‬,‫אף תשובה פה לא נכונה‬
10. The best source of iron for a child < 1 year?
a. Breastfeeding
b. Cow milk
c. Syrup ‫דניאל‬

11. Vitamin d dosage in children <10kg


a. Should be weight adjusted
b. 400 ‫דניאל‬
c. 600
d. 400-600
e. 1000
last exam- 11.2.22 ‫מבחן‬
What is NOT characteristic for constitutional delay of growth and puberty?
a. Normal size at birth
B. Short stature in a childhood , but relatively a normal adult height
C. Children tend do have delayed pubertal development compared to their peers, but is followed by catch-up when
they do enter puberty
D. Delayed skeletal age
E. Bone age consistent with the chronological age ‫מאור‬

What determines fetal growth? 212 ‫בספר הפולני עמ‬


a. Growth hormone- pubertal
B. Insulin
C. IGF - 1 ‫מאור‬
D. Sex steroids
E. Thyroid hormones- infant+ childhood

3. Incomplete GnRH - independent isosexual precocious puberty is NOT typical for :


A. McCune - Albright syndrome
B. testicular tumors
C. Septo-optic dysplasia spectrum- ‫מאור‬
D. congenital adrenal hyperplasia
E. estrogen-secreting neoplasm

According to the European Resuscitation Council guidelines during Pediatric Advanced Life Support procedures in
case of a shockable rhythm? present ,adrenaline should be administered :
A. as soon as possible
B. after 3 minutes (‫)למה לא זה? איתי‬
C. After 3 shock
C. after third at a dose of 10 mg / kg of body mass
E. no correct answer

What are frequent signs of nonclassic 21-OHO ,


a. Acne
B. Hirsutism
C. polycystic ovary disease
D. Irregular periods
E. All ‫מאור‬

According to the European Resuscitation Council guidelines, the Pediatric Basic Life Support procedures should be
used for: ‫ג שתיהן מופיעות בגיידליינס אז כל אחד והחלטתו‬+‫תשובות א‬
A . children up to 18 y . o . excluding newborns up to 4 week old ‫דניאל‬,‫ יום‬28 ‫מבצעים החייאת יילוד עד גיל‬
B. children up to 18. y . o . excluding infants
C. children Age 0-18 y . o . excluding newborns at birth - ARI (‫)זה הנוסח המדוייק של ההנחיות האירופאיות‬
D. children with an approximate body weight of up to 40 kg since over this value guidelines for adults should be used
E. all children age 0-18 y.o. with no exceptions

7. The approximate fluid loss in case of a moderate dehydration in children is


A. 10-20 ml / kg
B. 20-40 ml / kg
C. 45-60 ml / kg
D. 50-100 ml / kg
E. > 100 ml / lg
48. Which of the following symptoms are typical for moderate dehydration in children with acute diarrhea?
1) lack of thirst; severe
2) anxiety;
3) excessive thirst;
4) sleepiness;
5) skin goes back slowly after pinching.
A. 1,4
B. 2,3,5
C. 1,4,5 ‫פאכר‬
D. 3,4,5- ‫ הכי נכון) מסכימה מאור‬3+5 ‫ ליאל (אם היה‬-‫ יותר צמא‬,‫ככל שיותר מיובש‬
E. 3,4

In case of a severe dehydration which following symptoms are present:


A. normal breathing
B. tears are present
C. anuria
D. lowered blood pressure
E. C+D ‫מאור‬
63. Sign of slight dehydration is: ‫ ליאל‬-‫אף תשובה לא נכונה‬
a. Oliguria severe dehydration
b. dry tongue- moist
c. limited tears normal
d. capillary refill 2.5 s <2
e. skin fold straightens slowly normal
73. Physical findings consistent with dehydration don't include:
a. Increase pulse.
b. Increase blood pressure. → Lower
c. Increase capillary refill time.
d. Sunken fontanelle.
e. Decrease skin turgor.

52. Hemolytic uremic syndrome is diagnosed in the presence of the following symptoms :
A. nephrogenic anemia , thrombocytopenia , renal diabetes insipidus
B. haemolytic anemia , thrombocythemia , acute kidney damage
C. haemolytic anemia , thrombocytopenia , acute kidney damage ‫מאור‬
D. hyperbilirubinemia , liver failure , acute kidney injury
E. hyperbilirubinemia , thrombocythemia , acute kidney damage

53. The prognostic aspects of renal glycosuria are best described by the following statement: ‫מאור מהאינטרנט התשובה‬
a. The disorder does not require treatment and the prognosis for maintaining renal function is good
B. The disease is characterized by a high predisposition to urinary tract infections , which may lead to progressive
damage to the renal parenchyma
C. Constant renal glucose loss leads to episodes of severe hypoglycaemia and neurological complications
D. The prognosis is good as long as the drugs limiting glycosuria are used systematically
E. Glucose content in urine is a factor that strongly predisposes to the development of urolithiasis

17. The so-called Sutton's aphthas are a symptom characteristic of: ‫)י‬250( 263 ‫בספר הפולני עמ‬
A. Celiac disease- ‫ דניאל‬-‫ליאל מסכימה‬
B.Ulcerative colitis
C. Lesniewski - Crohn diseases
D. Exudative enteropathy
E. Peutz - Jeghers syndrome

18. The most common neoplasms in children aged 11-15 years include:
A. leukemia- ‫ליאל‬
B. Tumors of the brain
C.lymphomas
D. ft tissue sarcomas ?
E. tumors ?

Secondary bedwetting occurs after the so-called a "dry break " of at least :
A. 24 months
B.12 months
C. 12 weeks
D. 6 months
E. 6 weeks

9. For a child that weighs 15 kilograms the daily basic water demand is
a. 500 ml
B. 750 ml
C. 1000 ml
D. 1250 ml→(10*100)+(5*50)=1250ml/24h-‫שירן‬
E. 1500 ml

10. The scale for assessing the psychomotor development of a child aged 0-3 years is :
A Tarman and Merrill Scale
B. Wechsler Scale for
C. Brunet - Lezine scales The Scale of Psychomotor Development of Children (the Brunet-Lézine Scale)
D. COLUMBIA test
E None of the above

Fever lasting several hours with morning and evening fluctuations exceeding 1ºC is :
A. Continuous fever
B. Septic fever
C. Hectic fever.
D. intermittent fever- type or pattern of fever in which there is an interval where temperature is elevated for several hours followed by an interval
when temperature drops back to normal. This type of fever usually occurs during the course of an infectious disease
E. Dual - term fever- ‫אולי זה? יש חום שנקרא דאבל ובהגדרתו יש שני פיקים ביום‬the gerbil‫ גם חשבתי על זה שאולי התכוונו לדאבל‬-‫ליאל‬

12. The hereditary diseases caused by dynamic mutations of repeated trinucleotide sequences include all of the
following, except: 147-148 ‫בספר הפולני עמ‬
A. Huntington's disease
B. Fragile X syndrome- X-linked dominant- trinucleotide repeat expansion in the FMR1 (fragile X mental retardation 1) gene. CGG
C. Myotonic dystrophy type 1- autosomal dominant- a CTG trinucleotide expansion in the DMPK gene.CTG
D. Friedreich's ataxia type 1- autosomal recessive- trinucleotide repeat leads to progressive neurodegeneration GAA
E .Becker muscular dystrophy- X-linked recessive- caused by mutations in the dystrophin gene-> dysfunctional protein product, with
subsequent degeneration of muscle fibers and progressive proximal muscle weakness.

13 After birth, umbilical vein transforms into: Obliterates after birth and forms the round ligament of the liver
A. Round Ligament of the liver- ligamentum teres
B. Down oval
C. Arterial ligament
D. Venous ligament
E Does not convert

14. Indicate the incorrect sentence regarding Phenylketonuria: impaired ability to metabolize the essential AA phenylalanine
A. An autosomal recessive inherited disease caused by a mutation in the PAH cone
B. Phenylalanine Hydroxylase deficiency disease intended an autosomal dominant manner
C. A disease leading to the accumulation of phenylalanine in the body , which is toxic to the CNS
D. disease included neonatal screening
E Treatment of the disease involves the mediate introduction of a phenylalanine restricted diet eart of the respiratory
system

The complications of cystic fibrosis on the part of the respiratory system Include: 531 ‫בספר הפולני עמ‬
A. Fibrosis of the lungs
b. Abscess of the pleura
C. Pneumothorax
d. Bleeding from the respiratory tract
E. All of the above

Congenital heart defects with conduction-dependent systemic flow include all of the following except
A. Rupture of the aortic arch- ARI, ‫דניאל‬
B. Coarctation of the aorta
C. Tricuspid atresia
D. Critical aortic valve stenosis
E. left heart hypoplasia syndrome dependent systemic flow

45. Erythema infectiosum is a common viral infection- 286 ‫בספר הפולני עמ‬
1. It is also known as a fifth disease- ‫נכון‬
2. The virus is transmitted by respiratory secretions and by blood product transfusions
3. There is no specific therapy other than supportive care .
4. The rash appears in three stages "a slapped check " rash with circumoral paller, followed by maculopapular, truncal
rash appearance then fades an central cleaning takes place, giving a distinctive lacy, reticulated rush- ‫נכון‬
A. 1
B. 2,3,4
C. 2,3
D. 3.4
E. 1,2,3,4- ‫ליאל‬

54. The typical clinical manifestation of urolithiasis is renal colic. In the event of an attack of renal colic, the most likely location of
the plaque seen on.. tests performed during the course of symptoms the lower group is:‫לא מצויין תשובות אבל אין את השאלה הזאת‬

51 Changes in general urine test that indicate the diagnosis of a urinary tract infection include:
A. Presence of nitrates and ketone bodies
B. Increased number of leukocytes and the presence of ketone bodies
C. Presence of nitrates and increased number of leukocytes ‫ ליאל מסכימה‬-‫דניאל‬
D. Only Increased number of leukocyte
E. increased number of leukocytes and nephrotic proteinuria

Bt Pediatrics Final 22.7.21


1. Which of the following is true about puberty in a normal female?
40. Which of the following is true about puberty MOST healthy female? ‫שתי שאלות תשובה אחת‬
a. Begins with menarche.
b. Occurs at Tanner 1‫ א‬or 2→only 2.
c. Begins with pubarche.
d. Begins with thelarche. ‫שירן‬
e. Occurs between 14 to 16 years.
A. occurs at Tanner 3
B. begins with acne

2. After few days of uncharacteristic prodromal flu-like symptoms, patient developed white spots on the buccal
mucosa (Koplik spots). In the 3-4 day exanthem appeared at the hairline and spread cephalocaudally involving the
palms and soles. Your diagnosis and action are:
a. Rubella. You provide only a supportive treatment.
b. Measles. You provide only a supportive treatment- Vaccine within 72 hours of exposure for susceptible individuals ‫שירן‬
c. Measles. You administer antibiotics immediately→ its a virus.
d. Erythema infectiosum. You administer antibiotics immediately.
e. Erythema infectiosum. You provide only a supportive treatment.

3. Genomic imprinting is involved in etiology of: ‫ תשובות‬2 ,‫איחוד שאלות‬


a. Turner syndrome- 45,X.
b. Neurofibromatosis type 1- mutation in the neurofibromin-1 (NF1) gene.
c. Prader-Willi syndrome- deletion occurs on chromosome 15- ‫שירן‬.
d. Down syndrome- trisomy 21.
e. Achondroplasia- single-point mutations.
A. Turner syndrome
B. Prader-Willi syndrome
C. Angelman syndrome
D. Down syndrome
E. B and C- ‫שירן‬

4. Indicate the true sentence regarding Gilbert's syndrome: ‫ תשובה אחת‬,‫איחוד שאלות‬
a. Jaundice and/or hepatosplenomegaly.
b. Most often it appears in infancy.
c. It usually goes with severe stomach pain.
d. Increased unconjugated (free) bilirubin concentration is observed→ ‫שירן‬
e. May cause hepatosplenomegaly.
A. It is more common in boys than girls
B. Occurs very rarely in the polish population

5. "Red currant jelly" stool (stool mixed with blood and mucus) is the later symptom of:
41. Stool mixed with blood and mucus ("Red currant jelly'stool) is the later symptom of:
a. Intussusception- ‫שירן‬
b. Hyperthrophy of the pyloric muscle.
c. Anal atresia.
d. Esophageal atresia.
e. High intestinal atresia.

6. Which of the following statement about autosomal recessive inheritance is TRUE:


a. Males and females are equally likely to be affected.
b. The abnormal gene is most likely to code for a structural protein.
c. Is excluded by an absent family history.
d. Consanguinity decreases the risk of the disorder.
e. The abnormal gene can be carried on the sex chromosomes.

7. A 3-year-old child with poor growth and serum IgA anti-transglutaminase antibodies probably has:
64. A 4-year-old boy with growth below 3 pc and serum IgA anti- transglutaminase antibodies probably has:
a. Celiac disease - ‫איחוד שאלות תשובה אחת‬
b. Cystic fibrosis
c. Food allergy
d. Hyperimmunoglobulinemia A
e. Liver disease.
A. Organomegaly

8. In the treatment of viral croup we use all of the following EXCEPT:


a. Nebulised adrenaline.
b. Inhalation of humidified air.
c. Corticosteroids nebulised.
d. Cefuroxime iv. → anti bacterial - ‫שירן‬
e. Corticosteroid systemic.

9. The diameter of induration after Mantoux test is measured:


a. 36h after injection.
b. 36-48h after injection.
c. 48h after injection.
d. 48-72h after injection. ‫שירן‬
e. 72h after injection.
5. Mantoux test: the tuberculin reaction >= 5 mm is classified as in:
A. HIV- positive persons- danielle ‫ מאור‬https://www.cdc.gov/tb/publications/posters/images/mantoux_wallchart.pdf
B. Injection drug users
C. Persons with no known risk factors for TB
D. Mycobacteriology laboratory personnel-
E. Recent arrivals from high- prevalence countries

10. Secondary diabetes may be due to:


a. Hypothyroidism.
b. Cushing syndrome. ‫שירן‬
c. Addison disease.
d. Hypopituitarism.
e. Precocious puberty.

11. The physical manifestations of fetal alcohol syndrome include:


a. Cleft lip and palette.
b. Hepatomegaly, hypotonia, and microphthalmia.
c. Hyperbilirubinemia, jaundice, and failure to thrive.
d. Microcephaly, short philtrum, and prenatal growth retardation- ‫ליאל‬
e. Macrocephaly and hypotonia.

12. Which of the following is the clinical sign of rickets:


a. Craniotabes. ‫שירן‬
b. Rapid growth.
c. Thickening of the knees and pelvis.
d. Shrinking at the costochondral junctions.
e. Calcification in kidneys.
25. A fully developed clinical picture of rickets consists of the following set of symptoms:
1) craniotabes;
2) widening of the metaphysis of the forearm and shank bones;
3) kyphosis within lumbar spine;
4) decreased muscle tone;
5) delayed dentition.
A. 1,2,3
B. 1,2,3,5
C. 4,5
D. only 4
E. all of the above

13. In turner syndrome the most common congenital heart disease is: (324) 337 ‫מהמצגת ומהספר עמ‬
a. An atrial septal defect
b. Coarctation of aorta- ‫ליאל‬
c. Tetralogy of fallot
d. Bicuspid atrial valve- ‫אם הכוונה פה לביקוספיק האורטיק וולב אז זה הנכון‬
e. Tricuspid valve stenosis

14. Aspiration pneumonia can be the complication of:


a. Esophageal atresia ‫שירן‬
b. Anal atresia.
c. Hirschprung’s disease.
d. NEC.
e. Lower intestinal atresia.

15. A 4-month-old infant is seen at home by his GP because of two days of rapid, laboured breathing and poor
feeding. He was born at 27 weeks’ gestation, birth weight 979g and was discharged at 3 months of age. On
examination he has a fever of 37.4oC and a respiratory rate of 60 breaths/min. His chest is hyperinflated with
marked intercostals recession. On auscultation there are generalized fine crackles and wheezes. On arrival at
the Emergency Department his oxygen saturation shows 88%. What is the most likely diagnosis?
a. Pneumonia.
b. Bronchitis.
c. Bronchiolitis. ‫שירן‬
d. Tracheoesophageal fistula.
e. Asthma.

16. “Butterfly rash” in the malar area of the face is characteristic for:
a. Systemic Lupus Erythematosus. ‫שירן‬
b. Scleroderma.
c. Juvenile Rheumatoid Arthritis.
d. Juvenile Dermatomyositis.
e. Henoch-Schönlein purpura.

17. Brushfield spots in iris (pigmented spots) are characteristic clinical manifestations of: 139 ‫בספר הפולני עמ‬
a. Turner syndrome.
b. Edwards syndrome.
c. Down syndrome- ‫שירן‬
d. Klinefelter syndrome.
e. Williams syndrome.

18. Which of the following rashes in childhood are itchy:


1. Varicella
2. Scabies
3. Molluscus contagious
4. Seborrhoiec dermatitis
5. Atopic eczema
a. 1, 2, 3, 4, 5
b. 1, 2, 3, 5
c. 1, 2, 5 ‫שירן‬
d. 1, 5
e. 1, 2

19. The correct term for the movement of stomach contents into the mouth without effort and without vomiting reflexes
is:
a. Regurgitation- ‫שירן‬
b. Rumination.
c. Dysphagia- swallowing difficulties
d. Odynophagia- pain on swallowing
e. None of above.

20. Most of the serum immunoglobulin of normal infant at birth are:


a. Immunologically inactive.
b. Derived from mother.‫שירן‬
c. Derived from infant thymus.
d. In the IgM fraction.
e. In the form of isohemagglutinins (anti-A and anti-B).

21. Mother comes to Your office with her 5 year old son. She tells You that the boy is restless during sleep and has
itching in and around the anus and around the perineum. During physical examination You notice the excoriation and
erythema of the skin of perineum. Your diagnosis and treatment are:
a. Enterobiasis. It is sufficient to use Mebendazole/Albendazole treatment only of a boy.
b. Enterobiasis. You should use Mebendazole/Albendazole for treatment of the all family members. ‫שירן‬
c. Ascariasis. It is sufficient to use Mebendazole/Albendazole treatment only of a boy.
d. Ascariasis. You should use Mebendazole/Albendazole for treatment of the all family members.
e. Scabies. This infection needs only supportive treatment (with antipruritus ointments).

22. The first line antimicrobial drug in otitis media is:


a. Erythromycin.
b. Clindamycin.
c. Azithromycin.
d. Amoxicilin. ‫ שירן‬-‫מהמצגת‬
e. Ceftriaxone.

23. A 14-year-old boy is noted to be jaundiced. He has recently returned from India where he was visiting relatives in
a rural village. The most likely cause of his jaundice is:
a. HAV- ‫ליאל‬
b. HBV
c. HCV
d. HDV
e. Non of the above.

24. A 2-year-old child presents with a 10-day history of fever, a heart murmur, bilateral nonexudative conjunctivitis,
swollen and erythematous lips and strawberry tongue with erythematous and edematous hands and feet and a
polymorphous rash on the face, trunks, and extremities. The most likely diagnosis in this patient is:
a. Kawasaki disease.‫שירן‬
b. Measles.
c. Viral upper respiratory tract infection.
d. Group A beta hemolytic streptococcal pharyngitis.
e. Fifth disease.
9. A 2-year-old male presenting with 5 days of fever up to 39 degrees C (102.2 degrees F). On the second day of
illness, he developed red lips and an erythematous maculopapular rash over his torso. By the third day of illness, his
conjunctivae were injected without exudates, his rash involved his extremities, and he developed a strawberry
tongue. On the fourth day of illness, he had edema to his hands and feet with a diffuse red-purple discoloration over
the palms and soles, His lips were now cracked and bleeding. He was noted to be irritable and fussy, with decreased
oral intake. What is the most probable diagnosis?
a. Scarlet fever
b. Kawasaki disease ‫מאור‬
c. Rubella
d. Rubeola
e. Roseola
10. What treatment will you offer to the patient from question 9?
a. Infusion of IVIG 2g/kg
b. Infusion of IVIG 1g/kg
c. Aspirin
d. A+C ‫מאור‬
e. B+C

39. In the diagnostics of Kawasaki disease, the clinically significant symptoms do not include :
A. fever lasting for more than 5 days
B. erythema of the hands and feet
C. bilateral conjunctivitis
D. lip swelling and cracking
E. pneumonia- ‫ליאל מסכימה‬
47. Major criteria of Kawasaki Disease are, except:
A. Fever
B. Polymorphus rash
C. Conjunctival injections
D. Artritis ‫ ליאל מסכימה‬-‫מאור‬
E. Oral mucosal changes and strawberry tongue
22. What are the diagnostic criteria for Kawasaki Disease?
a. Fever > 5 days
b. Conjunctival vascular injection
c. Erythema, cracking and bleeding of lips
d. Strawberry tongue, polymorphous erythematous rash
e. All of them- ‫מאור‬

25. In the duct dependant systemic circulation, for maintaining the patency of the ductus arteriousus we usually use:
a. Prostaglangin E- ‫שירן‬
b. Indomethacin--> closes
c. Penicilin.
d. Digoxin.
e. Acetaminophen.

26. A 14-year old boy is seen in the emergency room because of a 3 week history of fever between 38.3C and
38.9C, lethargy and a 6kg weight loss. Physical examination reveals marked cervical and inguinal adenopathy,
enlarged tonsils with exudate, small hemorrhages on the soft palate, a WBC differential showed that has 50%
lymphocytes(10% atypical), and a palpable spleen 2 cm below the left costal margin. Which of the following
conditions is the most likely diagnosis? ‫ תשובה אחת‬,‫שתי שאלות‬
60. A 13-year old Mark: 3 week history of fever, lethargy and weight loss, marked cervical and inguinal
adenopathy, enlarged tonsils with exudate, small hemorrhages on the soft palate. WBC differential showed that
has 50% lymphocytes(10% atypical), and a palpable spleen 2 cm below the left costal margin. Which of the following
conditions is the MOST likely diagnosis? 282 ‫בספר הפולני עמ‬
a. HIV.
b. Varicella virus.
c. Streptococcal throat infection.
d. Kwasaki disease.
e. Infectious mononucleosis- ‫שירן‬

27. The parents of a 14-year-old boy are concerned about his short stature and lack of sexual development. By
history, you learn that his birth weight and length were 3 kg and 50 cm, respectively, and that he had a normal growth
pattern, although he was always shorter than children his age. The physical examination is normal and his growth is
at 10thpc. A small amount of fine axillary and pubic hair is present. There is no scrotal pigmentation; his testes
measure 4.0 cm3 and his penis is 6 cm in length. In this situation, which of the following is the most appropriate
course of action?
a. Measure pituitary gonadotropin.
b. Obtain a computed tomographic (CT) scan of the pituitary area.
c. Biopsy his testes.
d. Measure serum testosterone levels.
e. Reassure the parents that the boy is normal- ‫ליאל אבל לא בטוחה‬

28. During assessment of newborn immediately after birth we value:


a. Pulse rate, breathing and muscular tone.
b. Skin color, pulse rate, breathing, muscular tone, reflex irritability. ‫מאור‬
c. Skin color, pulse rate, pupil reaction, breathing, reflex irritability.
d. Skin color, pulse rate, pupil reaction, breathing, muscular tone, reflex irritability.
e. Skin color, pulse rate, breathing, muscular tone.
29. A 14-year-old girl is brought to the office for follow-up regarding recently diagnosed polycystic ovarian
syndrome. The patient was initially examined because of hirsutism, amenorrhea, and virilization. These findings are
most likely due to the effect of which of the following agents?
a. Estrogen.
b. Follicle-stimulating hormone.
c. Luteinizing hormone.
d. Testosterone. - ‫שירן‬
e. Thyroid-stimulating hormone.

30. What genetic syndrome should be suspected in a newborn with a heart defect and a characteristic arrangement of
the hands: in the clenched fists the index finger and the fifth are superimposed on the remaining fingers?
a. Edwards syndrome. ‫שירן‬
b. Patau syndrome.
c. Turner syndrome.
d. Angelman syndrome.
e. Down syndrome.

31. The continuous "machine-like", also described as "rolling-thunder" and "to-and-fro" heart
murmur (usually from aorta to pulmonary artery) is characteristic to:
a. Atrial septal defect (ASD.
b. Coarctation of the aorta (CoA).
c. Patent ductus arteriosus (PDA)- ‫ליאל מסכימה‬
d. Mitral stenosis.
e. Aortic insufficiency (AI).

32. The warning symptom (so-called red flag) accompanying abdominal pain is/are not: 254 ‫ספר פולני עמ‬
a. Navel pain= ‫ כאב באזור הטבור (ודגל אדום הרחק מאזור הטבור)י‬-‫ איתי ליאל‬-‫מסכים‬
b. Diarrhea at night.
c. Joint pain ‫ ארטריטיס יכול להיות סימן לצליאק או מחלת מעי דלקתית‬- .?‫מאור אולי זה התשובה‬
d. Delaying puberty.
e. Fever of unexplained etiology.

33. Which of the following associations is incorrect:


a. Prader-Willi syndrome – muscular hypotonia.
b. Turner syndrome – lymphoedema of feet at birth.
c. Noonan syndrome – pulmonary stenosis.
d. Silver-Russell syndrome – normal prenatal growth. ‫שירן‬
e. Neurofibromatosis type 1 – café-au-lait spots.
79. Which of the following statements about Silver-Russell syndrome (SRS) is incorrect:
a. SRS children are born small for gestational age.
b. SRS is an epigenetic disorder.( 11‫ ו‬7 ‫)כרומוזום‬
c. SRS is characterized by relative macrocephaly.
d. Children with SRS often present with body asymmetry.
e. Children with SRS catch-up normal growth in early childhood.‫דניאל‬

34. Morning stiffness is a common clinical manifestation of: 507 ‫ספר פולני ע’’מ‬
a. Systemic Lupus Erythematosus.
b. Scleroderma.
c. Juvenile Rheumatoid Arthritis. ‫שירן‬
d. Juvenile Dermatomyositis.
e. Henoch-Schönlein purpura.
36. A 5-day-old neonate is brought to the emergency department by ambulance 20 minutes after he had sudden
onset of irritability, diaphoresis and profound dyspnea. The patient has not had fever or other symptoms of systemic
illness. He was delivered vaginally at term without complications. Temperature is 37.0oC, pulse rate is 200/min, and
respirations are 50/min. On physical examination, auscultation of the chest shows a grade 2/6 systolic ejection
murmur that is heard best at the left upper sternal border and radiates to the left interscapular area. Palpation of
the abdomen shows enlargement of the liver. Femoral pulses are absent bilaterally, and the lower extremities
appear somewhat cyanotic compared with the upper extremities. No other abnormalities are noted. Which of the
following is the most likely diagnosis?
a. Coarctation of the aorta- ‫שירן‬.
b. Diaphragmatic hernia.
c. Group B streptococcal sepsis.
d. Patent ductus arteriosus.
e. Tetralogy of Fallot.

37. in addition to physical examination, which one of the following is most useful in diagnosing congenital hip
dysplasia in the newborn:
a. Radiograph.
b. Ultrasound.‫מאור‬
c. CT scan.
d. MR.
e. Bone scan.

34. A 15-year-old boy with fever > 38 degrees, weight loss and malaise was admitted to Department of Pediatric. 10
days ago he had a dental procedure, in the physical examination you found splenomegaly, hypotension, a new heart
murmur. During the echocardiogram a vegetation on valve was observed. Which clinical condition you can suspect?
a. Infective endocarditis- ‫שירן‬
b. Scarlet fever
c. A congenital heart disease
d. Leukemia
e. Meningococcal sepsis

39. The atypical pneumonia is usually caused by: ‫מהמצגת‬


a. Streptococcus pneumoniae and Chlamydia pneumoniea.
b. Streptococcus pneumoniae and Heamophilus influenza.
c. Mycoplasma pneumoniae and Chlamydia pneumoniae. ‫דניאל‬
d. Haemophilus influenza and Escherichia coli.
e. Mycoplasma pneumoniea and Haemophilus influenza.

40. The surface of the chest where the aortic valve is auscultated:
a. Left second intercostal space on the upper left sternal border.
b. Left third intercostal space on the left sternal border.
c. Left fourth, fifth intercostal spacer on the lower left sternal border.
d. Left fifth intercostal space on the left midclavicular line.
e. Right second intercostal space on the right sternal border ‫דניאל‬

41. During physical examination of a young girl, the examiner observed the external sex characteristics: Breast with
no glandular tissue, areola follows the skin contours of the chest. Small amount of long, downy pubic hair with slight
pigmentation on the labia majora. According to Tanner scale pubertal maturation can be described as:
a. M0, P0
b. M0, P1
c. M1, P2- ‫ ליאל מסכימה‬-‫דניאל‬
d. M1, P1
e. M1, P0

43. Cerebral fluid findings in acute bacterial meningitis are: 276 ‫בספר הפולני עמ‬
a. >100 leukocytes (usually thousands), protein 20-45mg/dl, normal glucose concentration.
b. >20 leukocytes, protein 20-45mg/dl, normal glucose concentration.
c. 100-500 leukocytes (monocytes predominate later), protein >100 /dl, depressed glucose concentration.
d. >100 leukocytes (usually thousands), protein 20-45mg/dl, glucose concentration in CSF usually <40 or <40% of
serum glucose. ‫דניאל‬
e. 100-500 leukocytes (monocytes predominate later), protein >100 /dl, glucose concentration <50%.

44. the plantar reflex (known as the Babiński response or Babiński sign) physiologically disappears:
a. Around 12 to 24 months of age- ‫ליאל‬
b. Around 12 to 24 years of age.
c. Around 6 to 12 months of age.
d. Around 6 to 12 years of age.
e. Around 12 to 24 weeks of age.

45. The clinical features of anorexia nervosa are:


1. Fine hair on the face and trunk (lanugo-like hair)
2. Rough and scaly skin
3. Tachycardia→ bradycardia
4. Hypothermia
5. Decreased body mass index
6. Erosion of enamel of teeth
7. Jaundice
a. 1, 2, 4, 5, 6 → ‫שירן‬
b. 1, 2, 4, 5, 7
c. 1, 2, 3, 5, 6
d. 2, 3, 4, 5, 6
e. 1, 3, 4, 7, 6

46. True contraindication for vaccines is:


A. Family history of seizures
B. Jaundice of the newborn
C. Infectious diseases in the incubation period
D. Convalescent phase of illness
E. Severe allergic reaction after a previous vaccine dose or to a vaccine component

47. Which of the following statements about acute glomerulonephritis is TRUE:


a. Spontaneous resolution is usually observed.
b. The most common cause in children is streptococcal infection.
c. After initial oliguria, polyuria with electrolytes loss may occur.
d. All of above.
e. None of above

48. The first clinical sign of nephrotic syndrome is:


a. Rash.
b. Edema.
c. Fever.
d. Hematuria
e. Proteinuria

49. Each of following agents has been associated with congenital infections EXCEPT:
a. Rotavirus- ‫ליאל מסכימה‬
b. Toxoplasma gondi.
c. Rubella virus.
d. Cytomegalovirus.
e. HIV.

50. The first attempt to breastfeed should be:


a. As soon as possible after delivery.
b. Not until the baby has been weighed.
c. Not until after at least an hour after delivery.
d. Not until after at least 24 hours after delivery.
e. Not until a first feeding of formula or sugar water has been administered to replenish the baby's strength.

51. BCG (Bacillus Calmette-Guérin) vaccination is used in many countries as a part of their tuberculosis control
programs because:
a. BCG provides protection against tuberculous meningitis and miliary tuberculosis in infants and young children up to
five years of age. ‫ ליאל מסכימה‬-‫דניאל מאור‬
b. BCG provides protection against primary pulmonary tuberculosis.
c. BCG provides protection against tuberculosis infection in patients who are severely ill or immunocompromised
d. BCG is used in tuberculosis treatment simultaneously with anti-TB drugs.
e. BCG efficacy does not wane over time and most vaccine-induced immunity does not change over time.

52. Which is not an early symptom of diabetes?


a. Polydipsia.
b. Polyuria.
c. Weight loss.
d. Enuresis (secondary.
e. Vomiting. ‫שירן‬

53. A four-year-old child, whose family follows a strict vegetarian diet, undergoes a prekindergarten physical
examination. The pediatrician is alert to the child's potential deficiency of:
a. Niacin.
b. Thiamin.
c. Vitamin B6.
d. Vitamin B12.
e. Vitamin C.

54. Which of the following are the clinical features of urinary tract infections in an infant or a young child:
a. Febrile convulsion.
b. Vomiting.
c. Recurrence of enuresis.
d. Abdominal pain.
e. All listed ‫ ליאל מסכימה‬-‫דניאל‬

55. A 5-week-old male infant who has forcefully vomited after almost every feed in the last
36 hours but continues to take his milk well. The MOST likely diagnosis is:
a. Intussusception- Intussusception- ‫ שבועות‬5 ‫ וזה מגיל חצי שנה ולא‬,‫יש ירידה באינטייק‬
b. Malrotation.
C. Pyloric stenosis- Pyloric stenosis. - ‫דניאל‬
d. Septicaemia.
e. Diabetic ketoacidosis.

56. Erythema infectiosum is caused by: 286 ‫בספר הפולני עמ‬


a. Epstein-Barr virus.
b. HSV-1.
c. HIV.
d. HSV-2.
e. Parvovirus B-19. - ‫שירן‬

57. A 3-year-old boy has been febrile and short of breath for 24 hours. There is a dullness on percussion with
reduced breath sound and lots of inspiratory crepitus over the lower lobe of right lung on auscultation. The MOST
likely diagnosis is:
a. Asthma.
b. Pneumonia. ‫דניאל‬
c. Pneumothorax.
d. Cardiac failure.
e. Bronchiolitis.

58. A 6-year-old girl has had vomiting and central abdominal pain for 2 days. She develops diarrhea. Examination
reveals mild dehydration, tachycardia of 110/min and tenderness in the lower abdomen with no guarding or rebound
tenderness, urine dipstick test reveals glucose(+++) and ketons(+++). What is the most likely diagnosis?
a. Diabetic ketoacidosis. ‫דניאל‬
b. Urinary tract infection.
c. Acute appendicitis.
d. Mesenteric adenitis.
e. Pyloric stenosis.

59. The patient has proteinuria, hipoproteinemia, hyperlipidemia, edema. What condition do you suspect?
a. Urinary tract infection.
b. Glomerulonephritis.
c. Henoch-Schonlein purpura.
d. Nephrotic syndrome.‫מאור‬
e. Acute renal failure.

60. You get a page at 6 am from the mother of the child suffering from type 1 diabetes mellitus. She says that the
child is having a seizure. You would advise the mother to do which of the following first:
a. Administer glucose tablets.
b. Give orange juice.
c. Administer glucagon. ‫שירן‬
d. Hold the insulin dose.
e. Increase the insulin dose.

61. During the diagnosis of cholestasis in a 6-week-old infant, the following tests should be performed:
a. Serological tests for TORCH infections.
b. General urine test and bacterial culture.
c. Research for metabolic diseases: GC-MS, alpha1-antitrypsin concentration.
d. Abdominal ultrasound examination.
e. All listed.‫דניאל‬
62. Wilms’ tumor is a neoplasm that typically occurs in children and its localization is in the:
a. Kidney.
b. Liver.
c. Suprarenal gland.
d. Urinary bladder.
e. Brain.

63. The sympoms and signs of early congenital syphilis can be:
a. Hepatosplenomegaly, jaundice, pancreatitis, hemolytic anemia. ‫מאור‬
b. Thrombocytopenia, petechial lesions, nerve deafness, saddle nose.
c. Clutton’s joints, interstitial keratitis, leukopenia, hydrops fetalis.
d. Rhinitis, bony lesions, mental retardation, rhagades.
e. Protruding mandible, flaring scapulas, petechial lesions, desquamations

64. The symptoms of Henoch-Schonlein purpura are:


a. Rash, abdominal pain, headache.
b. Nephritis, arthritis, rash, edema.
c. Abdominal pain, arthritis, nephritis.
d. Rash, abdominal pain, nephritis, arthritis.
e. Rash, proteinuria, hematuria, fever.
60. Choose the correct clinical evaluation in Henoch-Schonlein purpura:
A. Purpuric rash, hypertension, arthritis, abdominal pain
B. Vasculitis, nephritis, purpuric rash, heart failure
C. Purpuric rash, arthritis, abdominal pain, nephritis ‫מאור‬
D. Petechiae, diarrhoea, glomerulonephritis, proteinuria
E. Proteinuria, arthritis, purpuric rash, nephritis
39. Henoch – Schonlein purpura – we do not expect
a. Petechiae
b. Proteinuria
c. melaena
d. arthritis
e. Fractures ‫מאור‬

65. Choose pre renal cause of acute renal failure:


a. Vesicoureteral reflux.
b. Glomerulonephritis.
c. Hypotension- ‫דניאל‬
d. Henoch-Schonlein purpura.
e. Posterior urethral valves.

66. Which of the following is a cause of primary hypogonadism? ‫ שתי תשובות‬,‫ אותה השאלה‬-‫איחוד שאלות‬
a. Anorexia nervosa.
b. Turner syndrome.‫ דניאל‬-‫ליאל מסכימה‬
c. Craniopharyngioma.
d. Kallmann syndrome.
e. All above
A. Anorexia nervosa
B. Turner syndrome
C. Klinefelter syndrome
D. B+C ‫ לכן התשובה הן טרנר וקלנפיילטר‬,)‫ לא בהיפותלמוס\ היפופיסה(אז זה שניוני‬,‫ שחלות אשכים‬-‫ היפוגונדיזם ראשוני זה בעיה בגונדות‬-‫דניאל‬
E. All above
37. Which of the following associations is incorrect: 225 ‫בספר הפולני עמ‬
A. Kallmann syndrome - hypogonadotropic hypogonadism
B. Klinefelter syndrome - hypogonadotropic hypogonadism→ hypergonadotropic hypogonadism- ‫שירן‬
C. Turner syndrome - hypergonadotropic hypogonadism
D. Acquired gonadal damage – hypergonadotropic
E. All above are correct

67. The typical features of innocent heart murmurs include, except:


a. Soft, less than 3/6 in intensity.
b. Musical or vibratory in character.
c. Occurs during diastole- Most commonly midsystolic or continuous
d. Often position-dependent.
e. No palpable thrill.

2. Features suggesting that a murmur is significant are all below, except: ‫ תשובות‬2 ‫ אותם מסיחים‬,‫איחוד שאלות‬
a. Silent- ‫ליאל‬
b. Conducted all over the precordium
c. Thrill (equals grade 4-6 murmur)
d. Any diastolic murmur
e. Accompanied by other abnormal cardiac signs.
A. Loud
B. Any systolic murmur- ‫ליאל‬
65. All of the following suggest that the murmur is significant except:
a. A thrill- ‫נכון‬
b. Sinus arrhythmia on ECG
c. An additional soft diastolic murmur- ‫נכון‬

68. 12-year-old girl has a solitary thyroid nodule found on routine examination. She has no symptoms. Which of the
following is the most appropriate next step for this patient?
a. Fine needle aspirate.
b. CT scan of the neck.
c. Serum thyroid function tests
d. Trial of suppressive T4 treatment to look for nodule shrinkage.
e. Excisional biosy.

69. Which of the following neonatal features would support the diagnosis of cystic fibrosis: 331 ‫בספר הפולני עמ‬
a. Meconium aspiration.
b. Congenital pneumonia.
c. Delayed passage of meconium ‫מאור‬.
d. Hypoglycemia.
e. Eczema.
A. Precocious puberty

70. A 6-year-old boy with precocious puberty is more likely to have which of the following than a 6-year-old girl with
precocious puberty?
a. Neurological disorder.
b. Idiopathic precocious puberty.
c. Congenital adrenal hyperplasia.
d. Gonadal tumor.
e. Adrenal tumor.
71. Which of the following statement about the clinical examination of the child is TRUE:
a. The central cyanosis is best assessed by examining the nail bad- peripheral cyanosis
b. An enlarged spleen moves with respiration- ‫ליאל‬
c. The femoral pulses are reduced in patent ductus arteriosus- increased
d. Stridor is a low-pitched expiratory sound from distal airway obstruction. - high pitched
e. At birth the posterior fontanel is bigger than anterior fontanel- ‫להפך‬

72. Which of the following is the most frequent enzymatic defect in congenital adrenal hyperplasia?
56. The most common defect in congenital adrenal hyperplasia is: ‫ תשובה אחת‬-‫איחוד שאלות‬
a. 3 beta-hydroxysteroid dehydrogenase deficiency.
b. 11 beta-hydroxylase deficiency.
c. 21-hydroxylase deficiency. (95% of all CAH)
d. 22-lyase deficiency.
e. 17,20-hydroxylase deficiency.
A. 11B-hydroxylase deficiency
B. 3B-hydroxysteroid dehydrogenase deficiency
C. 17 a-hydroxylase deficiency
D. cholesterol deficiency

74. A child with cystic fibrosis requires all the following EXCEPT:
a. A high energy diet.
b. Regular chest percussion and postural drainage by parents.
c. Vitamin C-supplementation.Fat-soluble vitamin deficiencies.
d. Aggressive antibiotic treatment of chest infections.
e. Fat-soluble vitamins supplementation

76. A 18-month-old girl is still very unsteady on her feet. She tends to fall to her left side. On physical examination
increased tone and brisk reflexes are found in the left upper and lower limb.
a. Upper motor neurone lesion.- ‫דניאל‬
b. Lower motor neurone lesion
c. Cerebellar lesion.
d. Basal ganglia lesion.
e. Internal ear lesion.

77. A 10-year old boy, complaining of a sore throat and has a mild fever. His throat is red, tonsils are red and swollen
with white patches. Select the most likely diagnosis:
a. Diphteria.
b. Acute epiglotitis.
c. Exudative tonsillitis ‫מאור‬.
d. Herpes simplex infection.
e. Viral croup.

78. Sacrococcygeal tumor in newborn histologically is usually:


a. Teratoma maturum- ‫ליאל‬
b. Teratoma immaturum.
c. Sarcoma.
d. Rhabdomyosarcoma.
e. Neuroblastoma.
80. Relaxation, CNS dependence, increased blood pressure, increased heart rate, depressed temperature are the
effects of…………………….addiction.
a. Alcohol.
b. Marijuana.
c. Cocaine.
d. Nicotine. ‫דניאל‬
e. LSD.
51. Increased heart rate and blood pressure, increased respiratory rate and temperature, dilated pupils, increased
sweating are physical findings that may suggest overdose of:
A. Sympathomimetic (e.g. cocaine, amphetamines) ‫דניאל‬
B. Opioid (e.g. morphine, codeine) ‫מאור‬
C. Anticholinergic (e.g. tricyclic antidepressants, antihistamines)
D. Sedative hypnotic (e.g. anticonvulsants, benzodiazepines)
E. None of the above
7. Increased heart rate, increased temperature, dilated pupils and reduced sweating are physical findings that may be
observed in overdose
a. Tricyclic antidepressants
b. Morphine
c. Codeine
d. Cocaine
e. Anticonvulsants

Additional Questions:
Sample Exam Questions: Pediatrics Written Exam
1. An afebrile 18-month-old presents to the emergency department with the sudden inability to move the right arm.
There is no history of trauma. The next step is to:
A) notify social services.
B) extend and externally rotate the right forearm.
C) x-ray the clavicle- ‫ליאל נוטה להסכים‬
D) do a skeletal survey.
E) order a serum calcium and phosphate level.

2. Scheuermann disease, also know as vertebral epiphysitis, results in:


A) thoracic scoliosis.
B) lumbar lordosis.
C) cervical kyphosis.
D) lumbar scoliosis.
E) thoracic kyphosis- ‫מאור‬

3. The bone age of a child is judged by the: 219 ‫בספר הפולני עמ‬
A) presence or absence of various ossification centers as compared to known standards.
B) radio-density of the long bones as compared to known standards.
C) width-length ratios of the wrist bones as compared to known standards.‫ ליאל מסכימה‬-‫מאור‬
D) chondral development compared to cortical thickening.
E) width-length ratios of the long bones as compared to known standards

4. A 10-day course of antibiotic therapy is indicated for children with:


A) sterile pyuria.
B) hemorrhagic cystitis due to the adenovirus.
C) bacterial colonization of the urine without obstructive uropathy.
D) recurrent urinary tract infections with a functionally and anatomically normal urinary tract.‫ דניאל‬,‫מאור‬
E) persistent trace proteinuria.

5. A 2½-month-old afebrile infant presents with a staccato cough, tachypnea and conjunctivitis. Auscultation of the
lungs reveals sparse, fine inspiratory rales throughout both lung fields. Chest x-ray reveals patchy infiltrates bilaterally.
Which of the following is most likely the etiologic agent?
A) Chlamydia trachomatis ‫מאור‬
B) Cytomegalovirus
C) Bordetella pertussis
D) Staphylococcus aureus
E) Ureaplasma urealyticum

6. A 2-year-old female presents with fever, swelling and tenderness of the right anterior cervical nodes. The nodes feel
firm. The most likely pathogen involved is:
A) Hemophilus influenzae- ‫לא‬
B) Epstein Barr virus.
C) Staphylococcus aureus.
D) Streptococcus pneumoniae.
E) Mycobacterium avium intracellulare.

7. A 3-week-old female has jaundice. The baby was the product of a normal pregnancy and delivery; birth weight was
3100 gm. The mother states that the baby has been feeding well since birth, but the stools have been getting
lighter in color during the past two weeks. On examination, the liver edge is palpable 3 cm below the right costal
margin. Total serum bilirubin is 9.8 mg/dL (167.5 µmol/L) with a direct bilirubin of 3.2 mg/dL (54.7 µmol/L). The most
likely diagnosis is:
A) breast milk jaundice. Unconjugated or Indirect Hyperbilirubinemia
B) neonatal hepatitis. Conjugated or Direct Hyperbilirubinemia
C) maple syrup urine disease.
D) ABO incompatibility. Unconjugated or Indirect Hyperbilirubinemia
E) glucose-6-phosphate dehydrogenase deficiency. Unconjugated or Indirect Hyperbilirubinemia

8. A barium swallow is negative for reflux. Of the following choices, which is considered the “gold standard” assisting
in the diagnosis of gastro-esophageal reflux?
A) a trial of prokinetic agents.
B) esophagoscopy.Esophagoscopy with biopsy for diagnosis of esophagitis
C) pneumocardiogram.
D) esophageal pH probe. Esophageal pH probe studies and barium esophagography in severe cases.
E) upper GI series.

9. A thriving 4½-year-old male is brought to the office due to fecal soiling and chronic constipation. History reveals that
his stool habits were normal until “potty training” was initiated. Height and weight are at the 75th percentile. Physical
examination reveals a mass in the lower left quadrant and a fecal-filled rectum. The next step in management is:
A) barium enema.
B) saline enema.
C) Epsom salt enema.
D) tap water enema.
E) rectal biopsy.

10. An 18-month-old female presents to the emergency department with marked respiratory distress associated with
inspiratory and expiratory wheezing and a barking cough. Her history reveals that she had an upper respiratory
infection for 2-3 days when she awoke from sleep with inspiratory stridor. Which of the following is most helpful in
differentiating this as croup rather than bronchiolitis?
A) High grade fevers
B) The child’s age of 18 months
C) Improvement in symptoms with an aerosol of racemic epinephrine
D) Biphasic wheezing The most common causes of recurrent wheezing are asthma and COPD->bronchospasms in the small airways
E) The history of a preceding URTI

11. An injectable vaccine that is a potential risk for egg-sensitive individuals is:
(A) oral polio vaccine (OPV)
(B) diphtheria, tetanus, pertussis (DTaP)
(C) inactivated polio vaccine (IPV)
(D) influenza- ‫שירן‬
(E) Haemophilus influenzae type b (Hib)

12. The etiologic agent responsible for most urinary tract infections is:
A) Proteus.
B) Klebsiella pneumoniae.
C) Escherichia coli. ‫שירן‬
D) Mycoplasma hominis.
E) Chlamydia trachomatis.

13. Cri du chat (cat cry) syndrome is associated with:


A) hip dysplasia and gonadoblastomas.
B) ambiguous genitalia in males, Wilm’s tumor.
C) microcephaly, epicanthal folds, micrognathia (A condition of congenital mandibular hypoplasia). ‘‫ מאור‬,‫דניאל‬
D) pyloric stenosis, wide-spaced nipples.
E) macroencephaly, webbed neck, skeletal malformations.

14. Early sexual maturation following radiation therapy to the brain:


A) is seen primarily in girls treated at a young age.
B) occurs with equal frequency among boys and girls.
C) is seen rarely and only after very high doses of radiation.
D) is seen primarily in boys treated at a young age. ‫מאור אני חושבת שזה נכון‬
E) has not been observed.

15. Hashimoto’s thyroiditis is:


A) not seen in children.
B) caused by exposure to excessive radiation.
C) associated with Turner and Down syndromes.
D) a frequent cause of congenital hypothyroidism.
E) an auto-immune disease of the thyroid.
22. Hashimoto thyroiditis is characterized by all the statements except:
A. it is an autoimmune disease
B. It may progress into hypothyreosis
C. Thyroid tissue is infiltrated with lymphocytic cells
D. It is more common in iodine deficient countries , ‫ מאור‬,‫דניאל‬
E. Hyperthyreosis may develop in a course of this disease

16. In leukemia, common sites of extramedullary involvement include the:


A) liver.
B) testicle.
C) lung.
D) heart.
E) kidney.

17. Leukemoid reaction with lymphocytosis is associated with:


A) eczema.
B) Kawasaki disease.
C) infectious mononucleosis.
D) epiglottitis.
E) neurosyphilis.

18. Of the following physical stigmata, which would be most characteristic of Turner syndrome? 142 ‫בספר הפולני עמ‬
A) increased span to height ratio marfan syndrome
B) metatarsus varus
C) low-set ears
D) webbed neck- ‫שירן‬
E) narrow chest with wide spaced nipples ‫הפוך‬

19. Purulent pericarditis in children is most commonly caused by:


A) Haemophilus influenzae or â-hemolytic streptococcus.
B) Staphylococcus aureus or Moraxella catarrhalis.
C) Streptococcus pneumoniae or â-hemolytic streptococcus.
D) Haemophilus influenzae or Staphylococcus epidermidis.
E) Haemophilus influenzae or Staphylococcus aureus.

20. The animal reservoir and corresponding vector for Lyme disease is:
A) deer/deer tick.
B) wood rat/flea.
C) mouse/deer tick
D) raccoon/flea.
E) skunk/deer tick.
2021
1. Laboured breathing, hyperinflated chest, hyperresonance and fine crackles in all zones are chest signs observed in:
a. Asthma
b. Croup -. ‫ מאור‬- ‫לפי נלסון זאת התשובה‬
c. Bronchiolitis- ‫ שירן‬-‫מצגת‬
d. Pneumonia
e. None of the above

3. Normal resting pulse rate in children 2-5 years is:


a. 60-100
b. 80-120
c. 95-150- ‫לפי המצגת‬
d. 110-160
e. 120-150
4. Normal resting pulse rate in children between 5-12 years old is about:
A. 110-160
B. 95-150
C. 60-100
D. 80-120- ‫לפי המצגת‬
E. 100-120

4. Shrug shoulders and turn head against resistance is part of physical examination of:
a. II cranial nerve visual acuity is tested using a Snellen chart for distance vision or a handheld chart for near vision; each eye is assessed individually, with
the other eye covered
b. V cranial nerve The corneal reflex
c. VIII cranial nerve Balance has no truly satisfactory test for assessment, Formal vestibular testing can be performed
d. X cranial nerve Testing of the gag reflex should not be performed unless bulbar impairment is suspected, as it is uncomfortable to the patient.
e. XI cranial nerve- accessory nerve - A cranial nerve that innervates the sternocleidomastoid and trapezius muscles, thus controlling rotation of the head
and shrugging of the shoulders. It is the only cranial nerve with neurons in the spinal cord.

5. What is the age limit when child can say 3-word sentences?
a. 10 months
b. 12 months
c. 18 months
d. 2 years
e. 2,5 years

6. Late (decompensated) clinical signs of shock are:


1) Tachypnea early
2) Tachycardia early
3) Decreased skin turgor early
4) Acidotic (Kussmaul) breathing
5) Bradycardia
6) Decreased urinary output early
a. 1+2+3
b. 1+2+3+5
c. 4+5- ‫ליאל‬
d. 4+5+6
e. 3+4+5+6
69. Late (decompensated) clinical signs of shock are:
A. Tachycardia early
B. Bradycardia
C. Confusion/depressed cerebral state
D. B+C
E. A+C

8. Which of the following are clinical features of Edwards syndrome?


a. Low birthweight, small mouth and chin, short stature, cardiac and renal malformations
b. Structural defect of brain, cleft lip and palate, polydactyly, cardiac and renal malformations Patau Syndrome
c. Short stature, neck webbing or thick neck, widely spaced nipples, renal anomalies. Turner syndrome
d. Short neck, hypotonia, epicathic folds, small ears, flat occiput and third fontanelle
e. Macrocephaly, macroorchidism, long face, prominent mandible

11. A 5-year old boy is seen with maculopapular lesions on the legs and buttocks. He complains of abdominal pain,
weakness, dry cough. He has recently recovered from a viral upper respiratory infection. Complete blood cell count,
coagulation studies, and electrolytes are normal. Microscopic hematuria is present on urine analysis. What is Your
diagnosis?
a. Henoch-Scheonlein purpura
b. Hemolytic uremic syndrome
c. Rubella
d. Roseola
e. Rubeola

16. A 3-week-old infant with several days of mild diarrhea and decreased oral intake presents with severe tachypnea,
cyanosis and lethargy. The initial oxygen saturation is 87%, breath sounds are clear throughout and the cardiac exam
is normal. Peripheral perfusion is poor (CR > 4 sec.) and the administration of 100% oxygen by face mask results in a
saturation of only 94%. A portable CXR is normal. Of the following tests, which is MOST likely to reveal a diagnosis?
a. ECG
b. ABG
c. Echocardiogram
d. RSV
e. Methemoglobin level

17. Which of the following is the most likely diagnosis in a 2-month-old infant with a 10 day history of mild cough, who
is in no distress in the ED and has a normal physical examination:
a. FB ingestion While FB ingestion is always important to consider, it is unlikely at this young age
b. Viral bronchiolitis Bronchiolitis (as opposed to simply "RSV" infection," which can be fairly asymptomatic) is a clinical diagnosis
characterized by tachypnea, retractions, abnormal breath sounds, and/or other signs of lower respiratory tract infection; a normal examination
is sufficient to rule this out as a significant possibility.
c. Bacterial pneumonia ‫ לפי האתר‬Similarly, a young infant with bacterial pneumonia should appear ill, be febrile, and/or have other have
other abnormal physical findings. Petussis, on the other hand, is often subtle in young infants (the most susceptible population) and is usually
associated with a normal examination in the ED.
d. Congenital airway anomaly A congenital airway anomaly is also possible but unlikely, particularly if no signs were present at birth and
examination in the ED is normal
e. Pertussis In addition, pertussis has become far more common (or at least more commonly diagnosed) in the United States since about
2005, reaching epidemic proportions in some states. As a result, testing for pertussis should be considered in young infants with a prolonged
cough, even if they appear to be asymptomatic at the time of evaluation
https://www.acep.org/how-we-serve/sections/pediatric-emergency-medicine/quizzes/respiratory-emergencies-quiz/

19. A 9-year old boy is brought to the pediatric ED by his mother because of fever to 103 degrees fahrenheit
associated with generalized malaise and fatigue. Upon further questioning, the mother reports the boy was seen by
this pediatrician last week for low-grade fevers associated with muscle and joint soreness thought to be likely be
caused by a flu-like viral illness. The patient’s past medical history is significant only for a heart murmur caused by “a
problem with one of his valves''. Vital signs are temperature of 39.6, Heart rate 145, respiratory rate 38, blood
pressure 102/56 and pulse oximetry 98%. Physical examination reveals a tired-appearing child with a loud systolic
murmur heard best over the right upper sternal border. What is the most common organism causing infective bacterial
endocarditis in children? ‫)י‬345( 358 ‫בספר הפולני עמ‬
a. Staphylococcus aureus
b. Viridans streptococcus- ‫ליאל‬
c. Streptococcus pyogenes (Group A streptococcus)
d. Staphylococcus epidermidis
e. Influenza.

20. Which of these is not a sign of complete airway obstruction?


a. Stridor ‫מאור‬
b. obtunded
c. unable to cry
d. unable to speak
e. Lethargic
22. Which of this is a sign of complete airway obstruction? ‫?אולי היה תשובה עם הכל‬
a. Obtunded
B. Unable to cry
C. Unable to speak
D. Lethargic

21. A healthy 7-year old child is brought to you to be evaluated because he is the shortest child in his class. Careful
measurement of his upper and lower body segments demonstrate normal body proportions for his age. Which of the
following disorders of growth should remain in your differential?
a. Achondroplasia- short stature with skeletal disproportion
b. Morquio disease- Short stature, with a very short torso
c. Hypothyroidism
d. Growth hormone deficiency- ‫ליאל‬
e. Marfan syndrome.

22. A 13-year-old comes to your office expressing concern about his height. He had first seen you a year prior for his
routine checkup and a preparticipation sports physical for soccer (4cm below 3rd pc). Now in the eighth grade, all of his
friends are taller than he is, and he is at a disadvantage in the soccer field playing against much larger boys. After
obtaining height information from his parents shown here, you order a skeletal bone age radiograph. Which of the
following results would allow you to assure him of an excellent prognosis for normal adult height?
a. A bone age of 9 years ‫דניאל‬
b. A bone age of 13 years
c. A bone age of 15 years
d. Being at the 50th percentile for weight
e. Being at the 3rd percentile for weight

66. A 10-month-old infant has poor weight gain, a persistent cough, and a history of several bouts of pneumonitis. The
mother describes the child as having very large, foul-smelling stools for months. Which of the following diagnostic
maneuvers is likely to result in the correct diagnosis of this child:
A. CT the chest
B. Serum immunoglobulins
C. TB skin test
D. Inspiratory and expiratory chest x-ray
E. Sweat chloride test→ CF→ ‫שירן‬

24. A 16-year-old girl, accompanied by her mother, is in your office for a well-adolescent visit. The mother asks about
drug and alcohol abuse. You explain that the warning signs of abuse include which of the following?
a. Excessive concern for weight and body configuration
b. Improved school performance
c. Recent changes from age-appropriate, acceptable friends to younger associates
d. Deterioration in personal habits, hygiene, dress, grooming, speech patterns, and fluency of expression
e. Improvement in relationships with adults, siblings, and authority figures

26. The most common cause of bronchiolitis is:


a. Respiratory syncytial virus (RSV)--> ‫שירן‬
b. Human Metapneumovirus
c. Parainfluenza
d. Adenovirus
e. Rotavirus

27. Which test should be performed in diagnosis of sepsis?


a. Blood culture
b. Urine culture
c. CRP, WBC count
d. Platelet count
e. All of them ‫מאור‬

28. Which of symptoms listed below may be not a complication of bacterial meningitis? 277 ‫בספר הפולני עמ‬
a. SIADH- ‫נכון‬
b. Brain abscess- ‫נכון‬
c. Learning disabilities and behavioral problems
d. Deafness and seizures- ‫נכון‬
e. Nephrotic syndrome- ‫ליאל‬-‫מנינגיטיס סיבוך של נפרוטיק‬

30. You are evaluating a 6-year-old boy who complains about rash and itching. Physical examination reveals burrows,
papules, vesicles, and pustules localized mainly to the web spaces between the fingers, flexor surfaces of the arms
and wrists. The rest of the physical examination is normal, and the boy is otherwise well appearing. What is Your
diagnosis and treatment?
a. Rubella. You provide only a supportive treatment
b. Enterobiasis. You should use Mebendazole/ Albendazole for treatment of the all family members
c. An uncomplicated bacterial skin infection that require antibiotics
d. Scabies. The patient should be treated with topical scabicidal agents. The treatment is also recommended for
household members ‫מאור‬
e. Ascariasis. It is sufficient to use Mebendazole/ Albendazole treatment only of a boy

31. 10-year-old girl demonstrates the presence of antibodies against transglutaminase in the IgA class – 820
AU/ml (N<20AU / ml). What is Your next procedure? 263 ‫בספר הפולני עמ‬
a. Recommend gluten free diet- ‫לאחר אבחנה‬
b. Prepare bowel biopsy with histopathological evaluation- ‫ ליאל מסכימה‬-‫דניאל‬
c. Prepare the second general designation (IgAEmA, DPG)
d. Repeat test for antibodies against transglutaminase in 3 months
e. Calm down parents, recommend a low-lactose diet and control at the Gastroenterology Clinic next year.

32. Treatment with growth hormone and hormone replacement therapy will require most patients with:
a. Patau syndrome
b. Edwards syndrome
c. Down syndrome
d. Turner syndrome- ‫שירן‬
e. SIADH syndrome.

33. Features of the Down syndrome phenotype do not include:


a. diagonal lines- ‫שירן‬
b. Brushfield spots
c. Macroglossia
d. Microcephaly
e. muscular hypotension.
38. Features of the down syndrome phenotype include:
a. Brushfield spots
B. Microcephaly
C. Macroglosia
D. Muscular hypotension
E. all- ‫שירן‬

35. Definition of nephrotic syndrome includes:


1) Swelling
2) Ketonuria
3) Hypoalbuminemia
4) Daily protein loss > 3.5 g/1.73m2
5) Hematuria
a. 1,2,3
b. 2,4,5
c. 1,4,5
d. 2,3,5
e. 1,3,4
66. Clinical signs of the nephrotic syndrome are e.g.:
A. Periorbital oedema ‫ניראה לי רק זה מאור‬
B. Ascites
C. Breathlessness
D. A+B
E. A+B+C ‫דניאל‬
59. The symptoms of nephrotic syndrome are following:
A. Haematuria, hypertension, oedema, oliguria
B. Proteinuria, hypoproteinaemia, oedema, hyperlipidaemias ‫דניאל‬
C. Oedema, hyperkaliaemia, hyperlipidaemia, polyuria
D. Hypolipidaemia, proteinuria, hypoalbuminemia, oederna
E. Proteinuria, hyperproteinaemia, oedema, hyperlipidaemia
40. Nephrotic syndrome should not include:
a. Proteinuria
b. Hypoproteinemia
c. edema
d. hyperlipidemia
e. Hypergammaglobulinemia ‫ דניאל‬hypoalbuminemia

37. In a 2-month-old infant greasy and difficult-to-remove scale lesions have been noticed in the scalp. No signs of
itching, no pathological changes in other areas of the skin. Parents are healthy. The most probable diagnosis is:
a. Tinea capitis Round, pruritic scaly plaques with broken hair shafts or alopecia in affected areas
b. Seborrheic dermatitis ‫מאור‬
c. Atopic dermatitis intense pruritus and dry skin
d. Scabies
e. Psoriasis

38. Newborn presents bilateral conjunctivitis on the 30th day after the delivery. The next day it presents rhinitis, painful
cough and vomits. Additional findings: blood count normal, slightly increased CRP and interstitial pulmonary changes.
The most probable in the etiology of this condition is one of the following:
a. Staphylococcus aureus
b. Pseudomonas aeruginosa
c. Pneumocystis jirovecii
d. Chlamydia trachomatis ‫מאור‬
e. Toxoplasma gondii

39. Which of the following is not characteristic for a life-threatening anaphylactic reaction in children?
a. Urticaria
b. Dyspnea
c. Abdominal pain
d. Hypertension (shoul be HYpotention) ‫מאור‬
e. Mucous membrane swelling
45. Which of the following is characteristic for a life-threatening anaphylactic reaction in children?
Urticaria
Dyspnea
Abdominal pain
Mucous membrane swelling

40. Rubella infection in pregnant woman after 16 weeks of pregnancy may cause in newborn:
a. Eye malformations
b. Malformations of urinary tract
c. Hearing loss
d. Cataracts - ‫גם יכול להיות‬
e. Teeth abnormalities
2020
1. Which of the following viruses are most commonly associated with viral croup?
a. Adenovirus
b. Human papilloma virus
c. Varicella virus
d. Parainfluenza viruses
e. RSV

2. You are alone performing infant CPR. What is the correct ratio of chest compressions to ventilations?
a. 30 chest compressions to 2 ventilations
b. 15 chest compressions to 2 ventilations- for 2 ppl
c. 30 ventilations to 2 chest compressions
d. 15 ventilations to 2 chest compressions
e. only 30 chest compressions

3. In children, which of the following is the most common form of arrest?


a. Respiratory arrest ‫מאור‬- Respiratory arrest is the most common cause of cardiac arrest in children.
b. Ventricular fibrillation – cardiac arrest
c. Ventricular tachycardia – cardiac arrest
d. PEA – cardiac arrest
e. Intoxications

4. Characteristic pulmonary findings on physical examination include the following:


a. Tachypnea
b. Rales heard over the involved lobe or segment
c. Increased tactile fremitus, bronchial sounds, and egophony may be present if consolidation has occurred
d. Decreased tactile fremitus and dullness on chest percussion may result from parapneumonic effusion or empyema
e. All of the above ‫מאור‬

5. Which of these answers appropriately describes how to perform chest compressions on an infant?
a. Place two fingers on the sternum of the upper chest, one between the nipple line and the other 1 cm above
b. In two person CPR encircle infant’s torso with both hands with both thumbs pointing cephalic lying 1 cm below the
nipples over the sternum- ‫ ליאל מסכימה‬-‫דניאל‬
c. Palms placed in center of sternum above the nipples, using both arms
d. Palms placed 3 cm above the umbilicus
e. None of the above

6. Where should you palpate for a pulse on an unconscious toddler during CPR? ‫מהמצגת‬
a. femoral pulse
b. radial pulse
c. carotid pulse
d. brachial pulse - ‫ליאל‬
e. using stethoscope in 5th intercostal space

7. A 14-year-old girl awakens with a mild sore throat, low-grade fever, and a diffuse maculopapular rash. During the
next 24 hours, she develops tender swelling of her wrists and redness of her eyes. In addition, her physician notes
mild tenderness and marked swelling of her posterior cervical and occipital lymph nodes. Four days after the
onset of her illness, the rash has vanished. Which of the following is the most likely diagnosis?
a. Rubella- https://journals.co.za/doi/pdf/10.10520/AJA02599333_313‫מצאתי את השאלה עם התשובה דניאל‬.
b. Rubeola
c. Roseola
d. Erythema infectiosum
e. Erythema multiforme

8. A child can walk well holding on to furniture but is slightly wobbly when walking alone. She uses a neat pincer grasp
to pick up a pellet, and she can release a cube into a cup after it has been demonstrated to her. She tries to build a
tower of two cubes with variable success. She is most likely at which of the following age? 42 ‫בספר הפולני עמ‬
a. 2 months
b. 4 months
c. 6 months
d. 9 months
e. 1 year- ‫ליאל‬

10. The state laboratory calls your office telling you that a newborn infant, now 8 days old, has an elevated thyroid
stimulating hormone (TSH) and low thyroxin (T4) on his newborn screen. If this condition left untreated, the infant is
likely to demonstrate which of the following in the first few months of life: ‫)י‬466( 479 ‫בספר הפולני עמ‬
a. Hyperreflexia
b. Hyperirritability
c. Diarrhea- Constipation
d. Prolonged jaundice- ‫ליאל‬
e. Hyperphagia
11. clinical symptoms which are not typical for congenital hypothyroidism are:
a. Umbilical hernia
b. Prolonged jaundice
c. Hypertonia- ‫ליאל‬
d. macroglossia
e. constipation

12. An adolescent with type 1 diabetes returns for a follow-up visit after his annual checkup last week. You note that
his serum glucose is elevated, and his glycosylated hemoglobin (hemoglobin A1C) is 16.7%. This finding suggests
poor control of his diabetes over at least which of the following time periods?
a. 8 hours
b. 1 week
c. 1 month
d. 2 months- ‫שירן‬
e. 6 months

13. A 7-day-old boy is admitted to a hospital for evaluation of vomiting and dehydration. Physical examination is
otherwise normal except for minimal hyperpigmentation of the nipples. Serum sodium and potassium concentrations
are 120 mEq/L and 9 mEq/L (without hemolysis), respectively, serum glucose is 40 mg/dL. Which of the following is
the most likely diagnosis?
a. Pyloric stenosis
b. Congenital adrenal hyperplasia
c. Secondary hypothyroidism
d. Panhypopituitarism
e. Hyperaldosteronism

14. A 16-day-old infant presents with fever, irritability, poor feeding, and a bulging fontanelle. Spinal fluid demonstrates
gram-positive cocci. Which of the following is the most likely diagnosis?
a. Listeria monocytogenes
b. Group A streptococci
c. Group B streptococci <3 months
d. Streptococcus pneumoniae >3 months
e. Staphylococcus aureus
32. What are the bacterial causes of meningitis in the neonatal period?
a. Group B streptococci
b. Escherischa coli
c. Listeria monocytogenes
d. Enterococcus faecalis
e. All of above

15. While bathing her newly-received 2-year-old son, a foster mother feels a mass in his abdomen. A thorough
medical evaluation of the child reveals aniridia, hypospadias, horseshoe kidney, and hemihypertrophy. Which of the
following is the most likely diagnosis for this child?
a. Neuroblastoma
b. Wilms tumor- ‫ליאל‬- WAGR complex, Wilms tumour-aniridia syndrome
c. Hepatoblastoma
d. Rhabdomyosarcoma
e. Testicular cancer

16. Friends are considering adopting a “special needs” child from another country. The family has few details, but the
information they have received so far suggests the 4-year-old child has had surgery for an endocardial cushion
defect, is short for his age, and had a history of what sounds like surgically repaired duodenal atresia at birth. You
are suspicious this child may have the following syndromes? Subendocardial cushion defects are associated with Down syndrome.
a. Kleinfelter
b. Waardenberg
c. Marfan
d. Down- ‫שירן‬
e. Turner

17. A 15-year-old girl with short stature, neck webbing, and sexual infantilism is found of have coarctation of the
aorta. A chromosomal analysis likely would demonstrate which of the following?
a. Mutation at chromosome 15q21.1
b. Trisomy 21
c. XO karyotype- ‫שירן‬
d. Defect at chromosome 4p16
e. Normal chromosome analysis

18. A previously healthy 18-month-old has been in a separate room from his family. The family notices the sudden
onset of coughing, which resolves in a few minutes. Subsequently, the patient appears to be normal except for
increased amounts of drooling and refusal to take foods orally. Which of the following is the most likely explanation for
this toddler’s condition?
a. Severe gastroesophageal reflux
b. Foreign body in the airway
c. Croup
d. Epiglottitis The classic presentation: “three Ds” (drooling, dysphagia, and distress).
e. Foreign body in the esophagus

19. What does the initial resuscitation of hypovolemic shock include?


a. Infusion of isotonic crystalloids or albumin with boluses of up to 20 mL/kg crystalloids (or albumin equivalent) over
5-10 minutes- ‫שירן‬
b. Peripheral infusion of high-dose epinephrine
c. Peripheral infusion of high-dose norepinephrine
d. Hypertonic saline
e. None of them

20. Empiric antibiotics should be administered


a. 3 hours after arrival
b. Within 1 hr of the identification of severe sepsis
c. Within 2 hours of pressor initiation
d. After sampling the blood and urine culture- ‫שירן‬
e. It doesn’t matter

21. What are the 1st line antimicrobial drugs in sepsis in children aged >28 days?
a. Ceftriaxone + ampicillin- ‫שירן‬
b. Ceftriaxone + vankomicin 40 ‫ מצגת ספסיס שקופית‬.‫דניאל‬
c. Clarithromycin + penicillin
d. Vankomicin + amikacin
e. None of them

23. Please choose false sentence:


a. Children with asthma have symptoms of coughing, wheezing, shortness of breath, exercise intolerance V
b. During acute episodes, tachypnea, tachycardia, cough and prolonged expiratory phase may be present V
c. Inflammatory cells, chemical mediator and chemotactic factors mediate the underlying inflammation found in
asthmatic airways
d. Allergy skin testing are the elementary test for diagnosis of asthma ‫דניאל‬
e. Major indoor triggers for asthma are; viral upper respiratory tract, wood smoke, tabacco smoke and dust mites V

24. Which medication you do not use to treat asthma?


a. Inhaled corticosteroids
b. Leukotriene modifiers
c. Acyclovir- ‫שירן‬
d. Long-acting B2-agonists
e. Theophylline

25. Which of the following are not the risk factors for persistent asthma?
a. Atopic dermatitis, male gender, rhinovirus infections
b. Smoke exposure, pneumoniae, severe bronchiolitis
c. Asthma associated with obesity and early-onset puberty low birth weight
d. Using antimicrobial treatment at least three times per year-‫ ליאל מסכימה‬-‫דניאל‬
e. Peripheral blood eosinophilia > 4% (2-3 year of age)
26. What are the common causes of anaphylaxis in children?
a. Food, drugs
b. Latex
c. Insect sting or bite
d. Biologic agents
e. All of above ‫מאור‬

27. Which of the following is not caused by viral infection?


a. Measles
b. Rubella
c. Leptospirosis→ bacteria Leptospira- ‫שירן‬
d. Ebstein-Barr
e. Roseola

28. Choose untrue statement regarding the rubeola infection: 167 ‫בספר הפולני עמ‬
a. Rubeola is highly contagious and is caused by a single-stranded RNA paramyxovirus
b. The measles virus infects the upper respiratory tract and regional lymph nodes and is spread systematically during
a brief, low-titer primary viremia
c. Infection is utero results in significant morbidity from congenital rubella syndrome ‫דניאל‬.
d. Subacute sclerosing panencephalitis (SSPE) is a late neurological complication of rubeola infection with no
effective treatment - Subacute sclerosing panencephalitis (SSPE) is a late neurological complication of slow measles infection that is characterized by
progressive behavioral and intellectual deterioration and eventual death. There is no effective treatment.
e. The manifestations of the 3-day prodromal period are cough, coryza, conjunctivitis and the pathognomonic Koplik
spots

29. Koplik spots are:


a. Gray-white, sand grain-size dots on the buccal mucosa opposite the lower molars- ‫שירן‬
b. A characteristic transverse line of inflammation along the eyelid margin
c. Cervical lymphadenitis with brown rush on the neck
d. Rose-colored sots on the soft palate
e. Leukocytosis with an initial neutrophilic response
15. "Koplik spots" are clinical features of:
A. Rubella
B. Measles- ‫שירן‬
C. Kawasaki disease
D. Tuberculosis
E. Chickenpox

30. Please choose the right match:


a. Chickenpox – parvovirus B19 Varicella
b. Zoster – paromyxovirus
c. Rubeola – HHV-6 or HHV-7 measles virus (MV), an RNA virus of the Morbillivirus genus belonging to the Paramyxoviridae family
d. Scarlet fever – Staphylococcus aureus Group A Streptococci (e.g., S. pyogenes)
e. Infectious mononucleosis – EBV- ‫שירן‬

33. Which antimicrobial therapy (1st line treatment) you would recommended to treat pneumoniae caused by group B
streptococcus:
a. Penicillin or ampicillin iv, amoxicillin po- ‫שירן מסכימה מאור‬
b. Ceftriaxone iv
c. Cindamicin
d. Piperacillin – tazobactam
e. Aminoglycoside with ampicillin

35. Which medication you would recommend to treat hypothyroidism?


a. Hydrocortisone
b. Growth hormone
c. Levothyroxine- ‫ שירן‬synthetic form of T4
d. Thiamazole
e. Propranolol

36. 2-year-old child with severe diarrhea with mild signs of dehydration, parents are well cooperative. What is your
behavior?
a. You will recommend a high-calorie diet with a large volume of fat
b. You will recommend not feeding your baby for 6-8 hours
c. You will recommend to use oral glucose-electrolyte hydration and to return to normal diet,‫?דניאל מאור‬
d. You will recommend a lactose free diet
e. You will recommend a high protein diet to supplement the loss of protein in feces
Management of viral and most bacterial causes of diarrhea is primarily supportive and consists of correcting dehydration and
ongoing fluid and electrolyte deficits.

37. A 10-year-old patient reported to the doctor because of anorexia, abdominal pain with a decrease in body weight
of about 3 kg / 3 months, and recurrent aphthae in the mouth. What disease should you remember first?
a. ulcerative colitis - Manifests with multiple aphthae and pustules of the oral mucosa
b. intestinal tuberculosis
c. functional abdominal pain
d. Celiac disease- There is evidence of an association between gluten-sensitive enteropathy (celiac disease) and aphthous stomatitis
e. Food parasites

38. Gold standard of taking urine sample for urine culture with non-cooperating kid is:
a. Middle stream of urine
b. Clean catch
c. Catheterization ‫דניאל‬,Fakher
d. adhesive plastic bag
e. suprapubic aspiration ‫שקית שתן נועדה לקחת שתן לכללית ומיקרוסקופית ולא לתרבית‬.,‫פאכר‬,

41. Pyelonephritis – choose incorrect sign / symptom:


a. Dysuria Fever, costochondral tenderness, dysuria, urinary frequency, emesis
b. flank pain
c. chills, ‫מבוגרים פאכר‬/‫צמרמורת היא יותר מתאימה לבוגרים‬
d. vomiting
e. fever

31. The most common cancer in children in Europe is: ‫ בספר קורס‬388 ‫עמוד‬
A. Leukaemia ‫מאור‬
B. CNS tumours
C. Burkitt lymphoma
D. Wilms tumour
E. Neuroblastoma
42. What is the most frequent diagnosed cancer in pediatric population?:
a. Acute lymphoblastic leukemia- ‫ליאל‬
b. Retinoblastoma
c. Chronic myelogenous leukemia
d. Neuroblastoma ‫מאור‬
e. Wilms’ tumor
44. The MOST common extracranial solid cancer in childhood and the most common cancer in infancy is:
A. Hepatoblastoma
B. Wilms' tumor
C. Teratoma maturum
D. Neuroblastoma ‫מאור‬
E. Osteosarcoma
40. The most common solid malignant tumor in children is :
A. leukemia
B. primary bone tumor
C. brain tumor
D. liver neoplasm
E. soft tissue sarcoma
46. The most common malignancy in children is:
A. Brain tumor
B. Lymphoma
C. Pseudotumor cerebri
D. Leukemia ‫ ליאל‬-‫דניאל מאור‬
E. No answer is correct
2. The most common malignant tumor in newborns is:
A. Neuroblastoma ‫ ליאל מסכימה‬-‫מאור‬
B. Retinoblastoma
C. Nephroblastoma
D. Hepatoblastoma
E. Osteoblastoma

43. What is the most frequent location of neuroblastoma?


a. Paravertebral region
b. Suprarenal gland ??
c. Bones
d. Bone marrow
e. CNS

44. Which lymphadenopathy coexist with the greatest risk of neoplasm?


a. Supraclavicular region ‫מאור‬
b. Axillary
c. Submandibular
d. Cervical
e. Occipital

45. Indicate the right information about lymphadenopathy in children


a. Most lymph nodes are not palpable in a newborn infant
b. With varied antigenic exposure, lymphoid tissue increases in volume so that cervical, axillary and inguinal nodes
are often palpable during childhood
c. Lymph nodes are not considered enlarged until their diameter exceeds 1cm for cervical or axillary nodes and 1,5cm
d. All information are true ‘‫ מאור מידע מהאינטרנט‬,‫דניאל‬
e. All information are untrue

46. What is the presentation of acute leukemia in children?


a. Abnormal leukocyte and differential counts
b. Anemia
c. Thrombocytopenia
d. Fever of unknown origin
e. All mentioned above

62. Congenital heart disease with left-to-right shunt is/are:


a. ASD, VSD, PDA- ‫ליאל‬
b. Tetralogy of Fallot
c. AVSD
d. AS
e. Coarctation
47. Congenital heart disease with right-to-left shunt is/are:
a. ASD, VSD, PDA
b. Tetralogy of Fallot- ‫ליאל‬
c. AVSD
d. Coarctation
e. AS
63. TGA (transposition of the great arteries) is a congenital heart disease with: association with - VSD
a. Left-to-right shunt
b. Right-to-left shunt ‫דניאל‬
c. Common mixing
d. Outflow obstruction in a well child
e. Outflow obstruction in a sick neonate

48. Major manifestations of Jones criteria for diagnosis of rheumatic fever is/are:
a. Carditis 60%
b. Polyarthralgia
c. Erythema marginatum <5%
d. a + c
e. a + b + c
43. Revised Jones criteria are used in the diagnosis for the rheumatic fever in children. Which of the following is not a
major criterion?
a. arthritis.
b. Sydenham's chorea.
c. erythema marginarum.
d. subcutaneous nodules.
e. Fever.‫מאור‬

49. Mark the correct sentence


a. In streptococcus pharyngitis the drug of choice is penicillin- ‫שירן‬
b. The first-line treatment for sinusitis in children is doxycycline→ amoxicillin
c. The drug of choice in treatment of acute bronchiolitis in children are cephalosporins→ only supportive
d. A + B
e. A + B + C

50. The most common cause of chest pain in children is:


a. Musculoskeletal cause
b. Gastrointestinal cause
c. Cardiac cause
d. Respiratory cause
e. Zoster

51. Which of the followings are true?


1. measles rash has descending character→ true
2. thrombocytopenia may be the complication of rubella→ true
3. both chickenpox and zoster are caused by Varicella – Zoster virus;→ true
4. herpangina is caused by Herpes viruses→ coxsackieviruses
5. orchitis in the course of mumps concerns children after puberty→ true
a. 1,2,4
b. 1,4,5
c. 3,4
d. 1,2,3,5- ‫שירן‬
e. all of the above

52. Indicate false statement on Duchenne muscular dystrophy:


a. first symptoms occur between the 3rd-4th year of life DMD characterized by early childhoodnonset, typically within the first 5 years
b. it is characterized by symmetrical muscle atrophy that starts in the pelvic girdle and progresses centrally ‫מאור‬
c. pseudohypertrophy of the affected muscles occurs
d. deep reflexes persist ‫דניאל‬
e. awkward waddle and difficulties in getting up from a lying position and climbing the stairs are typical symptoms

53. Pyloric stenosis is characterized by:


1. the presence of hypertrophy of pyloric circular smooth muscle layer; pyloric muscle thickness >4 mm and length of pyloric canal >14 mm.
2. frequent occurrence in girls; more common in males
3. projectile non – billious vomiting; Initial symptom is nonbilious vomiting.
4. dehydration and metabolic acidosis; Hypochloremic, hypokalemic metabolic alkalosis is the classic electrolyte abnormalit
5. increased acetylcholinesterase activity on histopathology
a. 1,2,3
b. 1,3,4
c. 1,3 ‫ ש מטבוליק אלקלוסיס לא אסידוזיס‬,‫ לא נכון‬4 .‫דניאל‬.
d. 2,4,5
e. all of the above

54. Fifth disease is sometimes referred to as what?


a. “red tongue disease”
b. “swollen gland disease”
c. “spotted cheek disease”- ‫שירן‬ B19
d. “spotted palm disease”
e. “red eye disease”

55. Rubella infection during which trimester is most dangerous for the fetus?
a. First- ‫שירן‬
b. Second
c. Third
d. The risk is the same in every trimester
e. One week before the delivery

56. Which of these is not characteristic for the measles?


a. Maculopapular rash that appear with the fever peakV
b. Forschheimer spots on the soft palate Koplik spots ??
c. rash appears at the hairline and spread cephalocaudally involving
the palms and soles Spreads downwards, from behind the ears to the whole of the body.V
d. it is a highly contagious viral diseaseV
e. the infected person is highly contagious for 4 days before V
f. The rash appears until 4 days after the rash appears??
33. Which of these is characteristic for the measles?
a. Maculopapular rash that appears with the fever peak
B. Rash appears at the hairline and spread cephalocaudally involving the palms and soles
C. It is a highly contagious viral disease
D. The infected person is highly contagious for 4 days before the rash appears until 4 days after the rash spears

58. Which of these is not characteristic for the Allergic bronchopulmonary aspergillosis (ABPA)?
a. It occurs most often in people with asthma or cystic fibrosis V
b. Systemic antifungal therapy is not indicated
c. It is an allergic or hypersensitive reaction to a fungus Aspergillus fumigatus V
d. The first noticeable symptoms of ABPA are usually progressive worsening of respiratory symptoms such as
wheezing, cough, and shortness of breath
e. It is a clump of mold that colonizes pre-existing cavities in the lungs
.
59. Prerenal cause of acute kidney injury- choose wrong
a. Burns Prerenal
b. Hemorrhage Prerenal
c. Sepsis Prerenal
d. respiratory distress syndrome
e. Hemolytic – uremic syndrome-‫ שירן‬Intrinsic acute kidney injury

60. Most common causes of chronic renal failure in children choose wrong
a. Diabetes
b. reflux nephropathy
c. pyelonephritis- acute renal failure ‫איתי‬
d. glomerulonephritis
e. hypertension

61. Electrolyte abnormalities during chronic renal failure not include:


a. Acidosis
b. Hypocalcemia
c. Hyponatremia
d. Hyperkalemia
e. Hypophosphatemia ‫דניאל‬

62. Daily basic water demand for child 22 kg- 578 ‫בספר הפולני עמ‬
a. 1100 ml
b. 2200 ml
c. 1540 ml= (10kg*100ml)+(10kg*50ml)+(2kg*20ml)- ‫ליאל‬
d. 1900 ml
e. 1700 ml

64. The most common tumor in children that causes hypopituitarism is:
a. Prolactinoma
b. nonsecreting adenoma
c. pinealoma
d. meningioma
e. Craniopharyngioma ‫לפי קפלן‬

65. Gland X produces hormone B, which stimulates target tissue Y to produce hormone A if there is a congenital
receptor defect in target tissue Y causing hormonal resistance the blood hormone levels should be:
a. low A and low B
b. low A and high B - ARI- ‫ליאל‬
c. high A and high B
d. high A and low B
e. none of the above

66. Which of the following statements concerning steroid hormones is correct


a. They activate formation of second messenger
b. They are metabolized in the liver ‫ דניאל‬-‫ליאל מסכימה‬
c. They bind to cell membrane receptors
d. In the plasma they travel mostly in a free form
e. They are secreted in anterior pituitary

67. The most common cause of delayed puberty in boys is


A. hypogonadotropic hypogonadism
B. hypergonadotropic hypogonadism
C. Kallman’s syndrome
D. constitutional delay of growth and puberty- ‫שירן‬
E. craniopharyngioma

68. typical features of turner syndrome are:


a. cafe au lait spots
b. anosmia
c. hypergonadotropic hypogonadism
d. gastrointestinal tract abnormalities
e. none of the above

70. All of the following are chronic complications of diabetes mellitus except:
a. Retinopathy
b. Nephropathy
c. coronary artery disease
d. Hypoglycemia- ‫שירן‬
e. foot ulcers

2020-2
2. Acute community-acquired pneumonia causes include:
A. Streptococcus pneumonia
b. Mycoplasma pneumonia
c. Chlamydia pneumonia
d. Haemophillus influenza
E. All of the above ‫מאור‬

9. 7-year-old girl is brought to your office by her father because she has some acne, breast development, and fine
pubic hair. Which of the following is the most likely etiology for her condition?
a. A feminizing ovarian tumor
b. A gonadotropin-producing tumor- ‫שירן‬
c. A lesion of the central nervous system (CNS)
d. Exogenous estrogens
E. Early onset of "normal" puberty (constitutional)

10. Symptoms of pneumonia may be a bit different for each child. They may also depend on what is causing the
pneumonia. Cases of bacterial pneumonia tend to happen suddenly with these symptoms:
a. Cough that produces mucus, cough pain
b. Vomiting or diarrhea
c. Loss of appetite, tiredness (fatigue
d. Fever
e. all of them

29. Which of the following does not occur in the classic DiGeorge syndrome?
A. hypocalcemic tetany
B. malformed auricle Dysplastic ears
C. hyperthyroidism
D. increased susceptibility to infections
E. heart or major vessels defects
24. The clinical features of DiGeorge syndrome consists of:
a. absence or hypoplasia of the thymus gland
b. absence or hypoplasia of parathyroid glands
c. defect of cellular immunity
d. correct a + c
e. correct a+b+c- ‫ ליאל מסכימה‬-‫דניאל‬

34. Which of the following is considered in the pathomechanism of breastfeeding jaundice in newborns?
a. increased enterohepatic bilirubin circulation ‫מאור‬
b. delayed intestinal bacterial colonization
c. disorder of bile acids' metabolism.
d. increased lipoprotein lipase activity.
e. all the above mentioned.

36. In the fetus/newborn during hypoxia you expect one of the following, EXCEPT:
A. decreased pH
B. increased PaCO2
C. increased lactate level
D. decreased plasma K+ level
E. increased free fatty acids level

45. You will advice parents who have a child with gastroesophageal reflux:
a. AR milk.
b. HA milk.
c. only breast feeding.
d. amino acid mixtures.
e. soya milk.

48. Neutropenia is not characteristic of:


a. Evans syndrome.
b. Diamond-Shwachman syndrome.
c. primary immune neutropenia.
d. Kawasaki disease.
e. Kostmann disease.
79. Neutropenia is characteristic of: ‫?”אולי הסעיף האחרון הוא “כל התשובות נכונות‬
a. Evans syndrome
B. Diamond-Shwachman syndrome (SDS)
D. Primary immune neutropenia
E. Kostmann disease (Severe congenital neutropenia) – (SCN)

49. Acute life-threatening conditions that occur in pediatric oncology are:


a. tumor lysis syndrome V
b. Leukostasis. V
c. status epilepticus.
d. ileus.
e. all answers are correct. -ARI

50. The least probable finding in megaloblastic anemia of childchood is:


a. folic acid and/or Vit. B12 defficiency.
b. hepatosplenomegaly.
c. thrombocytosis.
d. psychomotor developmental delay.
e. lack of appetite, diarrhoea.
82. The most probable finding in megaloblastic anemia of childhood is:
a. Folic acid and/or Vit. B12 deficiency- ‫שירן‬
b. Thrombocytosis
c. Lack of appetite, diarrhoea (diarrhea)
d. Psychomotor development delay

51. The cause of hypoglycaemia in children could be, except for:


a. growth hormone deficiency.
b. excess insulin.
c. excess glucagon ‫ ’דניאל‬Fakher
d. hypothyroidism.
e. adrenocortical insufficiency.

52. The characteristic symptoms of metabolic syndrome include,


except for:
a. LDL above normal values and HDL below normal values.
b. lactose intolerance→ glucose intolerance
c. increase in blood pressure above normal.
d. fasting glucose above normal.
e. increased waist circumference or BMI above normal.

53. Choose the false statement concerning scarlet fever.


a. rash is caused by streptococcal pyrogenic exotoxin, V
b. It is related to the Group A beta-hemolytic streptococcal infection. V
c. the most intense lesions are found in the area around the mounth and naso-labial folds (Filatov triangle).
d. the rash forms the classic red streaks known as Pastia lines. V
e. form the second week desquamation (flakes peeling) appears. V

54. Medications used for treatment of constipation in children do not include:


a. Lactulose - V (‫)נורמלקס‬
b. Macrogol - V (‫)פוליאתילן גליקול‬
c. bisacodyl - V (‫)משלשל‬
d. Metoclopramide ( ARI ) - ‫ טיפול לגסטרופרזיס‬,‫ לא מטפל בעצירות‬,Fakher
e. Paraffin. - V

55. The anterior fontanelle usually closes:


a. by 2 months
b. by 1 year
c. between 4 to 18 months ‫דניאל‬
d. between 4 to 18 weeks
e. 3 years
27. Between which month of life does anterior fontanelle close?
A. 2-3
B. 4-6
C. 13-18- ‫ליאל‬
D. 20-24
E. 34-36

64. Which of the following statements concerning vasovagal syncope is false?


a. it is the most common type of syncope in children. accounting for 50 percent or more of cases presenting to the emergency department
b. in the majority of children directly before vasovagal syncope prodromal symptoms could be observed: dizziness
(vertigo). blurred vision, visual disturbances, fatigue. There is usually a prodrome of dizziness and light-headed feeling and abnormal vision
often with nausea, sweating, or pallor.
c. could be provoked by emotions or pain.
d. usually appears during physical effort. ‫רק זו תשובה לא נכונה פאכר‬, ‫דניאל‬.
e. vasovagal syncope prevention is to avoid stressful situations, increase intake of fluids and table salt.
102. Which of the following statements concerning vasovagal syncope is TRUE:
a. It is the most common type of syncope in children
b. In the majority of children directly before vasovagal syncope prodromal symptoms could be observed: dizziness
(vertigo), blurred vision, visual disturbances, fatigue.
c. Could be provoked by emotions of pain
d. Usually appears during physical effort

69. Which of the following is not absolute indication for prophylactic iron application?
a. prematurely born children.
b. infants of multiple pregnancies.
c. posthemorrhagic anemia.
d. heavy menstrual bleeding.‫ ליאל מסכימה‬-‫דניאל‬
e. infants of women with anemia during pregnancy.
104. Which of the following is an absolute indication for prophylactic iron application:
Prematurely born children
Posthemorrhagic anemia
Infants of women with anemia during pregnancy
Heavy menstrual bleeding

70. Which of the following below describe West syndrome:


a. Age of onset - 1-3 years
b. Tonic-clonic seizures in sleep, or simple focal seizures with awareness of abnormal feelings in the tongue and
distortion of the face
c. Violent flexor spasms of the head, trunk, and limbs followed by extension of the arms, last 1-2s- ‫ליאל‬
d. A+ B
e. A+ C
29. Which of following about West syndrome are correct? ‫ בספר קורס‬509 ‫עמוד‬
A. Age of onset: 3-12 months
B. Multiple seizure types, but mostly atonic, atypical absences, and tonic seizures in sleep
C. EEG: hypsarrhythmia
D. A+C ‫מאור‬
E. А+В+С

2019-1
1. The Gregg triad, which is a complication of intrauterine infection with the rubella virus, includes:
1. bone malformations
2. tooth deformations
3. cataract, glaucoma
4. deafness
5. heart defects
A. 1,2,3
B. 1,3,5
C. 2,4,5,
D. 1,3,4,
E. 3,4,5 ARI Fakher

3. Which interval should be maintained between giving the patient two live vaccines?
A. Any
B. 1 week
C. 2 weeks
D. 4 weeks at least 4 weeks
E. 6 weeks

4. According to the WHO recommendation, exclusive breastfeeding is recommended for the first:
A. 2-3 months of child’s life
B. 4-5 months of child’s life
C. 6 months of child’s life
D. 6-12 months of child’s life
E. 12 months of child’s life
WHO also recommends exclusive breastfeeding up to 6 months of age with continued breastfeeding along with appropriate
complementary foods up to 2 years of age or longer. Mothers should be encouraged to breastfeed their children for at least 1 year

5. The clinical reaction is considered to be an undesirable post-vaccination reaction (except for BCG) in the course of:
A. 24h after vaccination ‫מאור‬
B. 1 week after vaccination
C. 4 weeks after vaccination
D. 2 months after vaccination
E. 6 months after vaccination

6. Of the following, a a live vaccine is not:


A. BCG V
B. Chickenpox V
C. Rotavirus V
D. Viral hepatitis A Inactivated vaccine
E. Polio V
7. In the treatment of rotavirus diarrhea, the basis of the procedure is:
A. Administration of activated charcoal
B. Irrigation ‫ פאכר‬, ‫דניאל‬
C. Antibiotic from the group of beta lactams
D. A+B ‫מאור‬
E. B+C

8. Pathogens that often colonize the airways in children with cystic fibrosis are included:
A. Staphylococcus aureus
B. Haemophilus influenza
C. Pseudomonas aeruginosa
D. A+B
E. A+B+C ‫שירן‬-‫מאור‬
39 . Which of the following pathogens is the most frequent cause of respiratory tract infections in cystic fibrosis?
A. Staphylococcus aureus -‫ מאור‬-‫שירן‬
B. Streptococcus pneumoniae
C. Pseudomonas aeruginosa
D. Staphylococcus viridians
E. viruses

10. The rhythm for defibrillation is:


A. Tachycardia with narrow QRS syndrome
B. Ventricular tachycardia without pulse
C. PEA
D. Asystole
E. C+D

11. What part of the body is the intramuscular vaccine given to infants (under 1 year of age)?
A. Right arm
B. Left arm
C. Buttock
D. Thigh ‫מאור‬
E. Stomach
‫ פאכר‬,‫בלוטות לימפה צוואריות‬- ‫ אך מיקום הכי נפוץ‬,‫לא מבין את השאלה‬
12. The most common location of non-Hodgkin’s lymphoma is/are:
A. Head
B. Thorax -‫שירן‬
C. Abdomen
D. Marrow
E. Bones
The most common sites are in the chest, neck, or under the arms
13. In the diagnosis of cystic fibrosis, the following are used:
A. Screening test in newborns
B. Sweat test
C. Genetic test
D. Study of transepithelial potential difference
E. All of the above ‫מאור‬

14. Lymphoedema of the hands and feet of a newborn baby may be a symptom of: 138 ‫עמ‬
A. Down syndrome
B. Edwards syndrome
C. Turner syndrome- ‫שירן‬
D. Klinefelter syndrome
E. Prader-Willi syndrome

15. The most common cause of arrhythmias in children is:


A. Hypokalemia
B. Hypermagnesemia
C. Infections - ARI
D. Asphyxia
E. Malnutrition

16. A premature born infant with body weight 1200g has:


A. Extremely low birth weight X (<1000)
B. Low birth weight X (<2500)
C. Normal birth weight
D. Very low birth weight - ARI (1000-1500)
E. Small birth weight

17. In atypical pneumonia in children you will prescribe:


A. Antibiotics are not needed
B. Penicyllins
C. Cephalosporins
D. Aminoglicozides
E. Macrolides ‫מאור‬

18. In a 2 year old child with viral infection of the nasopharynx platelet counts is 770 thousands/ul:
A. Platelet count is within normal values and does not require investigation
B. This is reactive thrombocytosis and does not require investigation ‫ פאכר‬,‫דניאל‬
C. It is an indication for urgent haematological investigation
D. Antiplatelets drugs should be used
E. All answers are false

19. Factors predisposing to thrombosis are:


A. Polycythemia
B. Steroid therapy
C. Hypofibrynogenemia HYPER NOT HYPO Fakher
D. A+B ‫פאכר‬, ‫מאור‬
E. A+B+C

21. Which of the following is not the indication for colectomy in acute ulcerative colitis in children and adolescents?
A. massive hemorrhage
B. pseudopolyps - ARI (‫ לא אינדיקציה לפי‬UPTODATE)
C. bowel perforation
D. rapid course of ulcerative colitis
E. toxic megacolon

22. Ventricular tachycardia with the palpable pulse in a child with the presence of cardiac insufficiency symptoms is
the indication for:
A. defibrillation- (‫ שיש דופק עושים היפוך חשמלי מסונכרן(אם ניתן‬,‫דפיברילציה זה שאין דופק‬
B. rapid parenteral infusion of adenosine
C. cardioversion 100% ‫דניאל‬-
D. digoxin infusion
E. stimulation of the vagus nerve

23. Which of the following concerning procalcitonin (PCT) is true?


1) PCT under physiological conditions is released in very small quantities in the C cells of the thyroid;
2) in the course of an inflammatory response the increase of serum PCT is observed; the liver is then a major
source of PCT;
3) factor causing an increase in blood levels of PCT is mainly bacterial endotoxins;
4) PCT highest concentrations are observed in systemic viral infections;
5) PCT concentration increase 2-6 hours after the activation of the stimulus (early marker).
The correct answer is:
A. 1,4,5
B. 1,2,3,4
C. 1,2,3,5 Fakher
D. all of the above
E. none of the above

24. Which of the following concerning C-reactive protein (CRP) is true?


1) it is produced in the liver;
2) CRP binds to the bacteria, and then activates complement via an alternative route, taking part in the process of
opsonisation;
3) it plays an important role in the innate immune system capabilities directed against microorganisms, particularly
bacteria;
4) this parameter is non-specific;
5) it is an acute phase protein.
The correct answer is:
A. 1,4,5
B. 1,2,4,5
C. 1,3,4,5
D. all of the above - ARI , Fakher
E. none of the above

28. A General practitioner has to decide about the vaccination of a 13-month-old child against measles, mumps and
rubella. According to the parents rubella was diagnosed in this child by a doctor in the Emergency Room 2 months
ago. Choose the GP right decision:
A. the child can be vaccinated against measles, mumps and rubella at this visit in the GP’s office - ARI
B. the child can be vaccinated against measles, mumps and rubella in 4 weeks at the earliest
C. the presence of specific antibodies against rubella should be examined in the child’s serum and the vaccination
should be given only in the absence of specific antibodies
D. only monovalent vaccines against measles and mumps may be used in this child
E. the child can be vaccinated against measles, mumps and rubella in 3 years of the diagnosis of rubella at the
earliest

30. Which of the following does not occur in hemolytic uraemic syndrome?
A. microangiopathic hemolytic anemia V
B. leukopenia ‫ דניאל‬Fakher
C. thrombocytopenia V
D. fever
E. acute renal injury with oliguria V

31. Which of the statements on the rotavirus digestive tract infection in children is false?
A. water-mucous diarrhea without blood is the main presentation of the disease
B. it is frequently accompanied by fever
C. antibiotics are indicated in severe clinical cases ‫זה וירוס הטיפול הוא תומך‬
D. probiotics play important role in the treatment
E. secondary lactose intolerance may develop

33. In the treatment of Wilson’s disease you should administer:


A. gold salts
B. desmopressin
C. penicillamine ‫מאור‬
D. glucagon
E. copper salts Low-copper diet

35. Central motor neuron damage leads to:


1) spastic paralysis’ UMN lesion
2) flaccid paralysis; LMN lesion
3) loss of flexion;
4) increased deep reflexes;
5) muscle atrophy; UMN lesion (disuse atrophy) + LMN lesion (atrophy)
6) pathological reflexes. UMN lesion babinski +
The correct answer is:
A. 1,4,5
B. 2,3,6
C. 2,4,5
D. 1,3,6
E. 1,4,6

38. Which of the following are not considered as frequent causes of chronic cough in juvenile patients?
A. bronchial asthma
B. food allergy
C. cystic fibrosis
D. smoking ‫מאור‬
E. psychogenic

42. Which of the following are contraindicated in an acute asthma attack in children?
1) short acting beta2 agonists; Treatment of choice in the management of acute asthma
2) antitussives;
3) corticosteroids; Treatment of choice in the management of acute asthma
4) tranquilizers;
5) oxygen. Treatment of choice in the management of acute asthma
A. only 2
B. only 4
C. 1,2
D. 3,4
E. 2,4 ‫מאור‬

44. The most common cause of bacterial pneumonia in children is:


A. Streptococcus pneumoniae ‫שירן‬
B. Mycoplasma pneumoniae
C. Staphylococcus aureus
D. Klebsiella pneumoniae
E. Pseudomonas aeruginosa

47. 12-year old boy with a few-week history of skin rash which occurred in symmetric fashion over the
metacarpophalangeal and interphalangeal joints “lilac” rash on the upper eyelids and symmetric proximal muscle
weakness. The most probable diagnosis is:
A. Guillain-Barre syndrome
B. localized scleroderma
C. myasthenia gravis
D. childhood lupus erythematosus
E. dermatomyositis- ‫שירן‬

49. Incubation period of chickenpox lasts: 281 ‫ע’’מ‬


A. 10-23 days- ‫שירן‬
B. 3-5 days
C. 5-10 days
D. 20-30 days
E. 30-40 days

50. What is the frequency of splenomegaly in mononucleosis?


A. 10%
B. 20%
C. 30%
D. 40%
E. 50% 384 ‫נלסון‬

51. Hand, foot and mouth disease is caused by:


A. Human parvovirus B19
B. Human herpesvirus 6
C. Human herpesvirus 7
D. Enteroviruses ‫מאור‬
E. Cytomegalovirus

52. The mortality in Kawasaki disease is about?


A. 1-2% ‫דניאל‬
B. 3-4% ??
C. 5-6%
D. 10-12%
E. 15-20%

53. Which of the following statements about Lyme disease is incorrect?


A. Caused by spirochacte B. burgdorferi V
B. Infections occur most commonly in the summer v
C. The classical skin lesion in Lyme disease is known as erythema migrans v
D. Joint disease occurs in about 10% of cases Stage III (late Lyme disease)
E. The drug of choice for children below 12 years of age is amoxicillin v

54. The highest level of cortisol in a diurnal rhythm is found


A. at night
B. in the evening
C. at noon
D. in the morning
E. cortisol is not secreted in a diurnal rhythm

59. Which of the statements concerning type 1 diabetes is false


A. It is less common than type 2 diabetes mellitus
B. Treatment with insulin is necessary for survival
C. Complications are frequent
D. Hypoglycemia is rare- ‫ ליאל מסכימה‬-‫ פאכר‬,‫דניאל‬
E. Ketosis is common ??‫אולי זה מאור‬

60. Diagnostic criteria for diabetes mellitus is:


A. Fasting blood glucose 110-125 mg/dL
B. 2h plasma glucose in OGTT 140-199 mg/dL
C. Fasting blood glucose >140 mg/dL
D. Fasting blood glucose >126 mg/dL on two or more occasions- ‫שירן‬
E. 2h plasma glucose OGTT <200 mg/dL

19. The results of the OGTT test of a 12 - year - old boy without clinical symptoms showed :
- 105 mg / dL fasting glucose level
- 139 mg / dl in a 2 - hour test
How will you interpret the results ?
A. I can diagnose impaired fasting blood glucose and initiate metformin
B. I can diagnose impaired fasting blood glucose and initiate insulin
C. I can diagnose impaired fasting blood glucose and recommend a balanced diet with reduced simple sugars ‫דניאל‬
D. I can diagnose impaired fasting glucose and impaired glucose tolerance and refer to the Diabetes Outpatient Clinic
E. I will reassure the patient that his results are correct

61. Criteria for admission to the hospital children with acute asthma are:
A. No responded adequately clinically
B. Are becoming exhausted
C. Marked reduction in their predicted peak flow rate or FEV1 (<50%)
D. Reduced oxygen saturation (<92% in air)
E. All of the above

62. A symmetrical headache of gradual onset, often describe as tightness, a band or pressure, usually with no other
symptoms is a definition of:
A. Migraine without aura
B. Migraine with aura
C. Tension-type headache ‫מאור‬
D. Cluster headache
E. Mixed-type headache

63. Red flag symptoms of headaches are: ‫ בספר קורס‬504 ‫עמוד‬


A. Headache- worse lying down or with coughing and straining
B. Headache- wakes up child
C. Associated confusion, morning or persistent nausea or vomiting
D. Recent change in personality, behavior or educational performance
E. All of above

64. What are the high-risk features in acute lymphatic leukemia:


A. Age<1 year
B. Age >10 years
C. Tumour load >50 x 109/l
D. A+B+C
E. A+C
65. Peripheral neuropathy, haemolysis and ataxia are effects of deficiency: ‫ טבלה בספר קורס‬229 ‫עמוד‬
A. Vitamin K
B. Vitamin C
C. Vitamin A
D. Vitamin E
E. Vitamin D

67. When we check the capillary refill time we should apply blanching pressure for:
A. 2 s
B. 3 s
C. 5 s- ‫שירן‬
D. 8 s
E. 10 s

68. What is the dose of adrenaline during resuscitation of children; (ALS)


A. 10 ug/kg i.v.
B. 10 ug/kg i.o.
C. 100 ug/kg via tracheal tube
D. 0,1ml/kg of 1 in 10 000 soluble solution i.v.
E. all of the above- ‫שירן‬
26. The dose of adrenaline during ALS amounts to:
A. 1 ug/kg IV
B. 10 ug/kg IV ‫דניאל‬
C. 1 mg/kg IV
D. 10 mg/kg IV
E. 1 mg IV
70. All of the following are features of Prader-Willi syndrome except:
A. Failure to thrive in infancy
B. Hypotonia
C. Large feet small hands and feet
D. Hypogonadism
E. Uniparental disomy

2019-2
39. The following associations are characteristic of childhood malignancies, except:
A. acute myeloid leukemias and bleeding gums
B. neuroblastoma and raised urinary catecholamines
C. osteosarcoma and tumor in upper end of the femur ‫דניאל‬
D. hepatoblastoma and raised AFP (alpha-fetoprotein) in serum
E. brain tumors and early morning vomiting

40. Most common tumors diagnosed in teenagers are:


A. Osteosarcoma
B. Hodgkin lymphoma- ‫שירן‬
C. Neuroblastoma
D. soft tissue sarcoma
E. correct answer is A, B

41. Syndromes associated with Wilms tumor are:


A. WAGR
B. Beckwith-Wiedemann
C. Denys-Drash
D. Perlman
E. all answers are correct- ‫שירן‬

35. You notice that the patient has exophthalmos. What diagnosis would be most likely?
49. Exophthalmos is characteristic for: ‫ תשובה אחת‬-‫איחוד שאלות‬
A. Graves Basedow disease- ‫שירן‬
B. Hashimoto disease
C. Iodine deficiency
D. Congenital hypothyreosis
E. None of the above

50. Propiomelanocortin is a precursor of:


A. TSH
B. CRH
C. LH
D. ACTH- ‫שירן‬
E. GH
2018
11. Typical gonadotropins pattern in complete (true) precocious puberty is:
A. low FSH, high LH
B. low FSH, low LH
C. high FSH low LH
D. high FSH and high LH with FSH predominance
E. high FSH and high LH with LH predominance ‫מאור‬

14. Which of the following associations are correct?


1. B2-agonists – Salbutamol SABA
2. Anticholinergic – Theophylline Phosphodiesterase inhibitor
3. Oral steroids – Prednisolone corticosteroids
4. Anti-IgE monoclonal antibody – Montelukast LAMA
5. Inhaled steroids - budesonide
A. 1+3
B. 1+2+4
C. 1+3+5
D. 2+4
E. 1+2+3+4+5

18. Percentage of glycosylated hemoglobin provides information about:


A. amount of insulin needed to control postprandial hyperglycemia
B. the level of glycemia for the past three months ‫מאור‬
C. total blood ketones
D. hepatic glucose output
E. osmotic diuresis

20. Maternal rubella infection can be the cause of congenital cardiac disease:
A. Complete heart block
B. ASD
C. VSD
D. Peripheral pulmonary stenosis, PDA
E. Tetralogy of Fallot

21. Atrioventricular septal defect (complete) is characteristic for:


A. Turner syndrome→ coarctation of the aorta
B. Noonan syndrome
C. Down syndrome- ‫שירן‬
D. Klinefelter syndrome
E. Edwards syndrome

23. Every of the following are characteristic for hypothyroidism except:


A. Bradycardia
B. Decreased metabolic rate
C. Heat intolerance cold
D. Sleepiness
E. Weight gain

24. The most common cause of death in children between 15eyears to 19-years of age is:
A. Cancer
B. External causes including injury and poisoning
C. Circulatory system
D. Nervous system
E. Liver disorders

27. Which of the following statements about febrile seizures are incorrect? ‫ בספר קורס‬505 ‫עמוד‬
A. Affect 3% of children
B. Occur in children between the ages of 6 months and 6 years
C. There is a genetic predisposition, with about 25% risk if the child has a first-degree relative with febrile seizures
D. The seizure usually occurs early in a viral infection when the temperature is rising rapidiy
E. About 30-40% will have further febrile seizures

28. In generalized seizures, there is: ‫ בספר קורס‬508 ‫עמוד‬


A. Loss of consciousness if > 3 seconds duration
B. No Warming
C. Focal seizures ‫זה סוג אחר‬
D. A+B ‫מאור‬
E. A+B+C

33. What are the causes of acute liver failure in children >2 years:
A. Metabolic disease
B. Paracetamol overdose
C. Autoimmune hepatitis
D. в+с
E. A+B+C

34. Kayser-Fleischer rings are characteristics for:


A. Neroblastoma
B. Wilms tumour
C. Down syndrome
D. Fibropolycystic liver disease
E. Wilson disease-‫שירן‬

36. What is the limit age for walking?


6. What is the limit age for unsupported walking?
A. 10 months
B. 12 months
C. 14 months
D. 16 months
E. 18 months ‫מאור‬

37. When we check the vision and fine motor skills, a 7 months child can: ‫ בספר קורס‬42 ‫עמוד‬
A. Make marks with a crayon
B. Transfer toys from one hand to another ‫מאור‬
C. Built tower of three bricks
D. Draw without seeing how it is done
E. Draw after seeing how it is done

38. 12 months child can:‫ בספר קורס‬43 ‫עמוד‬


A. say 2-3 words other than 'dada' or 'mama' ‫מאור‬
B. say 6-10 words
C. show two parts of the body
D. join 2 or more words to make simple phrases
E. talk constantly in 3-4 word sentences

39. When about social, emotional and behavioral development, the child can hold spoon and get food safely to mouth
in the median age of:
What is the median age the child can hold a spoon and get safely to mouth:
A. 6 weeks
B. 6 months
C. 10 months
D. 14 months
E.) 18 months ‫מאור‬

40. Which of the following statements about autism spectrum disorders is incorrect:
A. The worldwide prevalence is estimated to be 7.6 per 1000 persons. 1000 ‫ מתוך‬14.7 ‫כתוב באמבוס‬
B. Presentation is usually between 2-4 years of the age. V
C. The child presents with a triad of difficulties and associated comorbidities
D. It is more common in girls ‫יותר נפוץ בבנים‬
E. Autism spectrum disorders are diagnosed by assessing the specific features and seeing if they meet a specific
threshold.

41. Which of the following associations is correct:


A. Hypermetropia - short sight long sight
B. Myopia - long sight nearsightedness that causes blurred distance vision
C. Nystagmus - abnormal curvature inventory repetitive an twitching movement of the eye
D. Astigmatism - rhythmical eye movement abnormal curvature of the cornea - blurred vision at all distance
E. Amblyopia – permanent reduction of visual acuity ‫מאור‬

42. The most common reasons for emergency admission of children under 15 years of age to hospital are:
A. Injuries and poisoning
B. Respiratory System Disorders ‫ מאור‬RSV?
C. Gastroenterological System Disorders
D. Neurological System Disorders
E. Urogenital System Disorders

44. The gross motor function level (functional ability) is described using the
Gross Motor Function Classification System (GMFCS). Level IV means:
A. Transported in a manual wheelchair
B. Self-mobility with limitations; may use powered mobility ‫דניאל‬
C. Walks using a handheld mobility device
D. Walks with limitations
E. Walks without limitations

45. An 8-month hospitalized child (9kg) is running a fever of 39,2C.


The correct drug, dose and route of administration to counter it would be:
A. Ibuprofen, 180mg, p.o.
B. Acetaminophen (Paracetamol), 125mg PR ‫ למה לא זה? פאכר‬,‫דניאל‬
C. No drug, only physical cooling
D. Ibuprofen 90mg, i.v
E. Acetaminophen (Paracetamol), 70mg, i.v.

48. Which of following statements about anaphylaxis in children/adolescents is incorrect:


A. Reaction is mainly to insects i.e. wasps (???)
B. Risk factors for fatal outcome include adolescent age group, coexistent asthma and nut allergy
C. Acute management is ABCDE and early administration of intravenous adrenaline
D. A+B
E. A+C ‫דניאל‬, C ‫ לא נכון כי תחילה המתן הוא לשריר‬Fakher
26. Which of the following statements about anaphylaxis in children/adolescents is CORRECT:
Risk factors for fatal outcome include adolescent age group, coexistent asthma and nut allergy
Acute management is ABCDE and early administration of intravenous adrenaline

49. The 2 years child opens eyes to pain, cries and localizes pain. How many will receive points in Children's Coma
Scale:
A. 8
B. 9 ‫מאור‬
C. 10
D. 11
E. 12

53. Which ofthe following associations is incorrect:


30. Which of the following are incorrect: ‫ תשובה אחת‬-‫איחוד שאלות‬
A. Edward syndrome - trisomy 18
B. Patau syndrome - trisomy 13
C. Down syndrome - trisomy 21
D. Turner syndrome - 45, X
Human parvovirus B19- Erythema infectisum
Human parvovirus B19- fifth disease (5th)
Epstein-Barr virus- mononucleosis (EBV)
All above are correct- ‫שירן‬
54. Maternal medications may adversely affect the fetus. What kind of adverse effect on fetus can have using SSRis
(selective serotonin reuptake inhibitors) during pregnancy:
A. Midfacial hypoplasia, central nervous system, limb and cardiac malformations, and developmental delay
B. Goitre, hypothyroidism
C. Congenital heart disease
D. Persistent pulmonary hypertension of the newborn ‫מאור‬
E. Enamel hypoplasia of the teeth

55. Which of the following statements about pyloric stenosis is incorrect:


A. In pyloric stenosis, there is hypertrophy of the pyloric muscle causing gastric outlet obstruction
B. It is more common in boys (4:1)
C. It presents at 4-8 weeks of age It usually presents during the third to fifth week of life.
D. Clinical features are vomiting, hunger after vomiting, weight loss
E. A hyperchloremic metabolic alkalosis with a high plasma sodium and potassium occurs as a result of vomiting
stomach contents- ‫ דניאל‬.‫יש היפוקלמיה היפוכלורמיה‬

56. Which of the following statements about Meckel diverticulum is correct: 238 ‫ספר קורס עמוד‬
A. Occurs in 20% of individuals around 2%
B. Generally asymptomatic, but may present with bleeding or intussusception or volvulus
C. Treatment is by surgical resection
D. B+C ‫מאור‬
E. A+B+C

57. The most frequent cause of gastroenteritis in children is:


A. Rotavirus
B. Norovirus 247 ‫ לפי הספר קורס עמוד‬fakher agree
C. Adenovirus
D. Coronavirus
E. Astrovirus

58. What is the most common cause of bacterial meningitis in the age of 1 months to 6 years:
A. Group B Streptococcus ??
B. Escherichia coli
C. Streptococcus pneumonia ??
D. Neisseria meningitides ?? ‫פאכר‬,‫ הייתי מסמן ניסיירה‬,‫מכיוון שקובצת הגיל רחבה‬
E. Haemophilus influenza

59. What etiology of meningitis could you suspect when the investigation of CSF shows:
- Appearance: Turbid
- White blood cells: Polymorphs, 200/mm
- Protein: 1,5 g/l
- Glucose: 12% of blood
A. Viral
B. Bacterial ‫מאור‬
C. Tuberculosis
D. The investigation result is correct
E. None of the above

60. Which of the following associations is incorrect:


A. Human parvovirus B19 - erythema infectiosum
B. Human parvovirus B19 - fifth disease
C. Epstein-Barr virus – mononucleosis
D. Cytomegalovirus – mononucleosis ‫מאור‬
E. B + D

62. Which of the following statements about Kawasaki disease is incorrect:


23. Which of the following information about Kawasaki disease is incorrect: ‫ שתי תשובות‬-‫איחוד שאלות‬
A. Mainly affects children of 6 months to 14 year, with a peak at the end of the 4 years life ‫ מאור‬Kawasaki disease
mainly affects children of 6 months to 4 years of age, with a peak at the end of the first year of life
B. The diagnosis is made on clinical features: fever over 7 days and 4 of the 5 other features V
C. Complications - coronary artery aneurysms and sudden death Cardiovascular signs: gallop rhythm, myocarditis, pericarditis
Coronary aneurysms (or other vessels) Sudden death V
D. Treatment - intravenous immunoglobulin and aspirin
E. A+B
A. Kawasaki disease is a systemic vasculitis mainly affects infants and young children
B. The etiology of Kawasaki disease remains unknown
C. The diagnosis is made on clinical features
D. The coronary arteries are affected in about two-third of affected children within the first 6 weeks of the illness
40. Which of the following statements about Kawasaki disease is CORRECT:
Complications- coronary artery aneurysms and sudden death
Treatment- intravenous immunoglobulin and aspirin

63. A medical student working in the emergency department sees a female baby, born 2 weeks ago, who is brought in
by her anxious mother. The mother tells the student that her baby seems "purple," especially her fingers and toes, and
looks extremely blue when crying. On physical examination, the sleeping baby has mild cyanosis of the face and
trunk, but moderate cyanosis of the extremities. Which of the following is the most common cause of cyanosis within
the first few weeks of life?
A. Atrial septal defect
B. Patent ductus arteriosus
C. Tetralogy of Fallot ‘‫ דניאל‬,‫פאכר‬The most common form of cyanotic CHD in the postinfancy period
D. Transposition of the great vessels The most common cyanotic heart lesion in the newborn period.
E. Ventricular septal defect

64. A 2-year-old child is hospitalized for evaluation of poor growth and low muscle tone. The
most striking physical finding is unruly, "kinky" hair, but the child also has increased joint
laxity and thin skin. Which of the following laboratory findings is most likely?
A. low ceruloplasmin ‫דניאל‬
B. High serum copper
C. Low serum iron
D. Low saturation of transferrin
E. Low serum haptoglobin

65. A previously well 1-year-old infant has had a runny nose and has been sneezing and coughing for 2 days. Two
other members of the family had similar symptoms. Four hours ago, his cough became much worse. On physical
examination, he is in moderate respiratory distress with nasal flaring, hyperexpansion of the chest, and easily audible
wheezing without rules. The most likely diagnosis is:
A. Bronchiolitis ‫מאור‬
B. Viral croup
C. Asthma
D. Epiglottitis
E. Diphtheria
68. A 3-year-old boy's parents complain that their child has difficulty walking. The child rolled, sat, and first stood at
essentially normal ages and first walked at 13 months of age. Over the past several months, the family has noticed an
increased inward curvature of the lower spine as he walks and that his gait has become more "waddling" in nature. On
examination, you confirm these findings and also notice that he has enlargement of his calves. This child most likely
has:
A. Occult spina bifida
B. Muscular dystrophy ‫מאור‬
C. Brain tumor
D. Guillain-Barré syndrome
E. Botulism

69. An 8-year-old boy is found to have progressive corneal vascularization, deafness, notched incisors, and a
flattened nose. The most likely cause of these changes is congenital infection by:
A. Toxoplasma
B. Rubella
C. Cytomegalovirus ‫מאור‬
D. Herpes simplex virus
E. T. pallidum (syphilis) ‫ זה תיאור של‬.‫ דניאל‬late congenital syphilis , ‫פאכר‬

71. A 13-year-old girl presents for evaluation of short stature (<10th percentile) and absence of breast development.
Her mother reports that the patient was the same height as her peers until a few years ago. On physical exam, patient
is found to have appropriate pubic hair and normal external female genitalia. There were no other significant findings
except for elevated blood pressure in both arms and weak femoral pulses. What is the patient's most likely cardiac
defect?
A. Tetralogy of Fallot
B. Epstein's anomaly
C. Patent ductus arteriosus
D. Coarctation of the aorta , Turner’s syndrome ‫פאכר‬
E. Eisenmenger's syndrome

2018 – 2
1. Which combination is incorrect?
A. Neonate < 4 weeks A neonate is a baby who is 4 weeks old or younger. A newly born infant is also called a neonate.
B. Infant < 1 year Infants (0-1 year of age)
C. Young child (preschool) 2-5 years
D. Infant < 2 years
E. School-age child 6-11 years

3. Rapid, shallow breaths, dull percussion, crackles and bronchial breathing during auscultation are chest signs of:
A. Pneumonia ‫מאור‬
B. Bronchiolitis
C. Asthma
D. Croup
E. Pneumothorax

5. Which cranial nerve do you check, if you ask the patient to close eyes tight, smile, and show teeth?
A. V
B. IX
C. VII -Facial nerve
D. I
E. XII

16. The most important part of pediatric BLS is:


A. High quality CPR ‫פאכר‬
B. Using AED as soon as possible
C. Transporting the child to the hospital immediately
D. Ventilation and preventing cardiac arrest
E. Everything is the same important

17. Which of the following statements about PALS is incorrect: ‫מהמצגת‬


A. VF and pulseless VT are shockable rhythm V
B. Energy of 1 defibrillation should be 4.J/kg V
C. After defibrillation immediately resume CPR for 1 min for 2 min
D. Give adrenaline every 3-5 min epinephrine
E. Consider amiodarone after 3 and 5 shocks

30. Which diseases could be prevent by vaccination?


A. Measles
B. Rubella
C. Tetanus
D. Rabies
E. All of the above ‫מאור‬

31. In the Apgar Score we assess:


- 1. Heart rate
- 2. Respiratory effort
- 3. Muscle tone
- 4. Reflex irritability
- 5. Colour of the skin
A. 1 + 2
B. 1+2+3
C. 1+2+4
D. 1+2+3+4
E. 1+2+3+4+5

32. Acceptable pre-ductal SpO2 10 min after birth is


A. 90% Fakher
B. 80%
C. 70%
D. 60%
E. 50%
94. Acceptable pre-ductal SpO2 5 min after birth is:
80% Fakher
95. Acceptable pre-ductal SpO2 3 min after birth is:
70% Fakher

33. Orchidometer is used to assess:


A. Bone age
B. testicular volume ‫מאור‬
C. Height
D. weight
E. Ovarian volume

34. The first sign of female puberty is:


A. Pubic hair grow
B. Menarche
C. Palpable breast bud ‫מאור‬
D. Axillary hair growth
E. Height spurt

35. Genetic causes of short stature include all of except:


A. Turner syndrome
B. Russell-Silver syndrome
C. Marfan syndrome→ Long-legged tall stature- ‫שירן‬
D. Nooman syndrome
E. C+ D

36. Premature sexual development is defined when secondary …. Characteristics occurs before:
A. 7 years of age in female
B. 7 years of age in males
C. 8 years of age in females ‫מאור‬
D. 8 years of age in males
E. 9 years of age in females

38. Which of the following statements about delayed puberty is incorrect


A. Is more common in males V Delayed puberty is common in boys and is usually due to constitutional delay of growth and puberty.
Delayed puberty is uncommon in girls and a cause should be sought.
B. Most commonly, this is constitutional delay in growth and puberty V
C. It may not be induced by dieting or excessive physical training May respond to nutritional supplements
D. A + C
E. В + С

39. The advantages of breastfeeding for the mother are:


- 1. Promotes close attachment between mother baby
- 2. Increases the time interval between children, which is important in reducing birth rate in developing countries.
- 3. Reduces risk of breast cancer
- 4. Reduces risk of ovarian cancer
- 5. Reduces risk of type 2 diabetes
A. 1 + 2
B. 1+2+3
C. 1 + 2 + 3 + 4
D. 1+2+3+4+5
E. 1+2+4

43. Which of the following examples are 'Red flag' symptoms or signs in the children with constipation:
- 1. Failure to pass meconium within 24 hours of life
- 2. Faltering growth
- 3. Gross abdominal distension
- 4. Sacral dimple above natal cleft, over the spine
- 5. Abnormal appearance
- 6. Perianal bruising or multiple fissures
- 7. Perianal fistulae, abscesses or fissures
A. 1+ 2+3+4
B. 1 + 2 + 3 + 7
C. 2+3+4+5+7
D. 1 + 2 + 3 + 4 + 5 + 7
E. 1+ 2+ 3+4+5+6+7

49. Interferon-gamma release assays (IGRAs) are blood-based tests for:


A. Tuberculosis ‫מאור‬
B. Kawasaki disease
C. Rubella
D. Mononucleosis
E. Herpangina

50. The way of tuberculosis transmission is (choose the correct answer):


A. congenital infection
B. inhalation of air-borne droplet nuclei containing M. tuberculosis ‫ לא מועבר בדרך אחרת‬,‫בידוד טיפתי בלבד‬,‫פאכר‬
C. ingestion of unpasteurized milk or dairy products from some infected cattle
D. direct inoculation of infected material on through mucous membrane or wound through a break in the skin
E. all of the above ‫מאור ארי‬

51. Choose the correct answer about attenuated vaccines:


A. Are created by reducing the virulence of a pathogen, but still keeping it viable
B. Can be safely administered to people with weakened immune system
C. several doses (2 or more) must be administered to provide adequate and persisting antibody response.
D. Two or more attenuated vaccines can be administrated at 4-week minimum interval between doses (if not
administered simultaneously)
E. A and D ‫מאור‬

52. True contraindication for vaccination is NOT:


A. severe allergic reaction (eg, anaphylaxis) after a previous vaccine dose or to a vaccine component.
B. administration of live-attenuated vaccines in pregnant women
C. chronic diseases of heart
D. cerebral palsy
E. C and D ‫מאור‬
Rotavirus vaccine: severe combined immunodeficiency (SCID), history of intussusception; GI tract malformation (e.g., Meckel's diverticulum)

53. In which situation meningococcal prophylaxis iS NOT strongly recommended?


A. Your patient share eating utensils with the index patient.
B. Your patient perform mouth - to - mouth resuscitation on the index patient within 7 days before onset of illness
C. Your patient is medical personnel, who had indirect contact with index patient in the hospital department
without direct exposure to the index patient's secretions within 7 days before onset of illness.
D. Your patient (young child) had household contacts or nursery school contact with index patient within 7 days before
onset of illness
E. Your patient sat directly next to the index patient during airline flight lasting longer than 8 hours.

55. In which situation You should refer Your anorectic patient immediately to a hospital?
A. Patient developed bulimic symptoms.
B. You notice the signs of dehydration or arrhythmia.
C. You noticed acrocyanosis of hands and feet
D. During the current episode of anorexia nervosa, the patient has regularly engaged in binge-eating or purging
behavior
E. Patient with amenorrhea

61. A 36-hour-old full term newborn is having bilious emesis in the nursery after each feed, and he has not yet had
his first bowel movement. His prenatal, birth, and family history are unremarkable. His abdomen is mildly distended
with hypoactive bowel sounds, but his exam is otherwise normal including digital rectal exam. Abdominal X-ray
shows distended loops of bowel, as well as a ground-glass appearance in the right lower quadrant. What is the
MOST LIKELY underlying diagnosis in this infant?
A. Cystic fibrosis
B. Duodenal atresia ‫מאור‬
C. Hirschsprung disease
D. Meconium plug syndrome- ‫שירן‬
E. Small left colon syndrome

2017
2. First line antibiotics used in tuberculosis therapy are:
A. Isoniazid, Amikacin, Rifampicin, Pyrazinamid
B. Rifampicin, Pyrazinamide, Ethambutol, Kanamycin
C. Isoniazid, Amikacin, Kanamycin, Pyrazinamide
D. Isoniazid, Rifampicin, Pyrazinamide, Ethambutol- ‫שירן‬
E. Isoniazid, Rifampicin, Amikacin, Ethambutol

3. Treponemal tests used to detect anti treponemal antibodies are:


A. VDRL, RPR, FTA-Abs ‫מאור‬
B. MHA-TP, TPHA, FTA-Abs- ‫שירן‬
C. TPHA, RPR, FTA-Abs
D. VDRL, MHA-TP, TPHA
E. RPR, VDRL, MHA-TP

7. Which kind of Aspergillosis should NOT be treated by antifungal


drugs
A. Cerebral aspergillosis
B. Allergic bronchopulmonary aspergillosis (ABPA)
C. Aspergilloma -’, ‫ לדעתי זה נכון – לפי אמבוס לא תמיד‬,‫מסכימה דניאל‬
‫דורש טיפול מאור אם המטופל אסיפטומטי‬
D. Invasive aspergillosis
E. All kinds of Aspergillosis diseases should be treated by antifungal drugs

8. The most common complication of ascariasis is:


A. Pancreatitis
B. Cerebral encephalitis
C. Perforation
D. Appendicitis
E. Intestinal obstruction ‫מאור‬

16. The prevalence of anorexia nervosa in teenage girls is about: 449 ‫בספר הפולני עמ‬
A. 1,5%
B. 5% → ‫שירן‬
C. 10%
D. 25%
E. 42%
20. Symptoms and signs that suggest organic disease are:
- 1. Epigastric pain at night, hematemesis
- 2. Diarrhea, weight loss, growth failure, blood in stools
- 3. Vomiting - persistent
- 4. Jaundice
- 5. Dysuria, secondary enuresis
A. 1,2,3
B. 1,2,3,4
C. 1,2,3,5 ‫מאור‬
D. 1,2,4,5
E. 1,2,3,4,5

21. Which of the following associations is incorrect:


A. Aortic stenosis - murmur, upper right sternal edge
B. Pulmonary stenosis - murmur, upper left sternal edge
C. ASD - ejection diastolic murmur at upper left sternal edge
D. VSD - pansystolic murmur at lower left sternal edge
E. PDA - continuous murmur at upper left sternal edge continuous murmur at the left infraclavicular region

22. In tetralogy of Fallot, as implied by the name, there are four cardinal anatomical features EXCEPT:
A. A large ASD
B. A large VSD
C. Overriding of the aorta
D. Subpulmonary stenosis
E. Right ventricular hypertrophy

24. In rapid assessment of level of consciousness (AVPU) "P"


means:
A. Alert
B. Responds to voice
C. Responds to pain
D. The child's airway is at risk and will need to be maintained
by a maneuver or adjunct
E. C+D

25. Optimal position for chest compression in infant is:


A. Two thumbs on lower half of sternum
B. Two thumbs on upper half of sternum
C. One hand over lower half of sternum
D. One hand over upper half of sternum
E. Both hands over lower half of sternum

27. Advantages of breastfeeding for the infant are that is EXCEPT:


A. Is life-saving in developing countries ‫דניאל‬
B. Reduces the risk of gastrointestinal infection, otitis media and NEC ‫מאור‬
C. Enhances the mother-child relationship
D. Reduces risk of breast and ovarian cancer
E. Reduces the risk of insulin-dependent diabetes, hypertension and obesity in later life

28. Which of the following information about Pyloric stenosis are correct:
A. More common in girls It is more common in boys (4:1)
B. Sings are visible gastric peristalsis, palpable abdominal mass on test feed and possible dehydration
C. Associated with hyponatraemia and hypokalaemia A hypochloraemic hypokalaemic metabolic alkalosis develops as a result of
vomiting stomach content
D. A+B+C
E. В+С

31. All of the following statement about growth are true EXCEPT:
A. Growth of the long bones ceases with epiphyseal fusion Long bones lengthen at the epiphyseal plate with the addition of bone
tissue and increase in width by a process called appositional growth
B. Bone age is used to assess skeletal maturation
C. In females the growth spurt occurs after the menarche Growth spurt occurs 1 year after thelarche
D. Male have a greater pubertal growth spurt than female
E. The gain of body length is the biggest during the first year of life

33. Correct cardiothoracic ratio in teenager is:


A. 20,7
B. 21,0
C.=<0,5 ‫מאור מצאתי מהאינטרנט לא בטוה‬, Fakher
D. >=0,6
E. >5,0

37. 4y.o. child has a cough and a runny nose for 2 days, a temperature of 38,0 C. Physical examination reveals slight
cervical adenopathy, reddened tonsils with no exudate, rales above the lungs. Blood test shows WBC 14000/ul, NEUT
36%, LYM 60%, RBC 4,1 mln/ul, HGB 12,1 g/dl, CRP 13 mg/l. Your initial diagnosis and course of action would be?
A. Bacterial infection, hospitalization
B. Viral infection, symptomatic treatment at home ‫שירן‬,‫דניאל‬
C. Bacterial infection, antibiotic therapy at home
D. Viral infection, hospitalization ‫מאור‬
E. Viral infection, antibiotic therapy at home

38. Most common cause of acute abdomen in adolescent children is:


A. Acute appendicitis ‫מאור‬
B. Acute cholecystitis
C. NEC
D. Megacolon toxicum
E. Constipation

39. You happen to come across a collapsed child. A quick assessment shows no response, no signs of breathing, a
pulse of 60/min, pallor. The correct course of action would be:
A. Leave the child be.
B. Call for help, set the child in a safe position
C. Start CPR, call for help
D. Give rescue breaths, call for help
E. Check for foreign objects in the respiratory tract

42. Blood tests in the pyloric stenosis will NOT reveal:


A. HyperK hypokalemic
B. HyperCl hypocloremic
C. PH and aldosterone 1
D. HypoCl
E. A+B ‫מאור‬
62. Blood test in the pyloric stenosis will require:
a. Hypo K
b. Hypo Cl
c. pH ↑ and aldosterone ↑
d. none of them
e. all of them

50. A 3.5-year-old child with fever and short of breath for 24 hours. There is a dullness on percussion with reduced
breath sound and lots of inspiratory crepitus over the lower lobe of right lung on auscultation. The MOST likely
diagnosis is:
A. Asthma
B. Pneumothorax
C. Cardiac failure
D. Bronchiolitis
E. Pneumonia

51. All of the following statement about puberty are true EXCEPT: ‫ שתי תשובות‬,‫ אותה השאלה‬-‫איחוד שאלות‬
A. Breast development is the first sign in females
B. Pubarche is the first sign in males Testicular enlargement
C. A testicular volume of 2 ml is prepubertal
D. Some breast development may occur in normal pubertal males
E. A testicular volume of 5 ml is pubertal
a. Penile growth is the first sign in males ‫ דניאל‬-‫ליאל מסכימה‬
b. A testicular volume of 4ml is pubertal

55. All of the following sentences about palivizumab are true ЕХСЕРТ:
A. Is a monoclonal antibody against RSV
B. Reduces the risk of hospitalization due to RSV infection ‫מאור‬
C. Is indicated in premature infants
D. Is considered as a standard treatment for bronchiolitis - ‫ ליאל מסכימה‬-‫דניאל‬
E. Is indicated in infants with congenital heart disease

62. Choose the wrong answer about Abusive Head Trauma (AHT):
A. Results in the significant morbidity and mortality
B. The symptoms that strongly suggest AHT are: subdural hematomas, retinal hemorrhages and diffuse axonal injury
(especially when they co-occur)
C. Children always present severe external signs of injury ‫דניאל‬
D. Acute intracranial trauma is best evaluated via initial and follow- up CT
E. May be caused by direct impact, asphyxia or shaking

66. A 1.5-year-old Anna is still very unsteady on her feet. She tends to fall to her left side. On physical examination
increased tone and brisk reflexes are found in the left upper and lower limb. Which of the following best describes the
pattern of neurological signs?
A. Hemiplegia ‫דניאל‬
B. Diplegia
C. Atactic disorder
D. Spastic quadriplegia
E. Choreoathetoid cerebral palsy

67. Which formula would you use to calculate BMI?


A. kg/m
B. kg/m2 ‫דניאל‬
C. kg'/m
D. kg/cm2
E. m/kg2

69. Wheezes are sounds characteristic for:


A. Upper airway obstruction
B. Pneumonia
C. Croup
D. Lower airway obstruction - bronchitis?
E. Non of the above

72. Newborn screening for congenital adrenal hyperplasia is based on measurement of.
A. Skeletal maturation (bone age)
B. 17a-hydroxyprogesterone
C. DNA mutation analysis
D. Testosterone
E. Androstenedione

73. All of the following are problems commonly associated with cerebral palsy EXCEPT:
A. Epilepsy
B. Mental retardation
C. Blindness? ‫דניאל‬
D. Behavioral problems
E. Strabismus ?

74. The most useful diagnostic test for scarlet fever is the:
A. Rapid strep test or throat culture - ‫זה ה‬goldstander
B. Blood culture
C. Complete blood count and antistreptolysin titer
D. Monospot test or Epstein-Barr virus titers
E. All correct

78. The first deciduous tooth usually erupts:


A. Before 5 months
B. between 5 and 9 months ‫מאור‬
C. between 9 and 12 months
D. between 12 and 18 months
E. after 18 months
first visit to the dentist when the first baby (deciduous) tooth erupts, which is at about 6 months, and then on a regular basis
79. The line anterior fontanel usually feels closed on physical examination between 9 months and 18 months
A. by 3 months feels closed on physical
B. by 5 months
C. by 12 months
D. by 18 months ‫מאור‬
E. by 24 months

80. At an 8-week surveillance review you would expect an infant all ЕХСЕРТ:
A. smile responsively
B. quieten to certain sounds
C. fix and follow a moving face
D. grasp an object placed in his hand
E. roll over ‫ מאור‬from belly to back at 4 months

2016 A
18. Lesions Caused by scabies in infants are localized most likely on:
A. between fingers
B. waistline
C. web spaces
D. palms and soles- ‫ ליאל מסכימה‬-‫דניאל‬
E. palms but never soles

27. The following clinical features: patient is wearing oversized layered clothing to hide appearance, fine hair on the
face and trunk (lanugo-like hair), rough and scaly skin, bradycardia, Hypothermia, decreased body mass index,
erosion of enamel of teeth (acici from emesis), acrocyanosis of hands and feet are characteristic for:
A. Binge eating
B. Bulimia
C. Anorexia nervosa→ ‫שירן‬
D. Hypothyroidism
E. Cushing syndrome

30. What should You remember about during the interview with an adolescent patient
A. The questions should be appropriate for the patient's developmental age
B. Interviewing an adolescent alone and discussing confidentiality to obtain the information regarding adolescent
risk-taking behaviours
C. To use STEP guide, HEADDSS Topics and confidentiality guidelines
D. Conversations with parents and adolescent are confidential, but there are some exceptions, f. ex. abuse, suicide,
homicide
E. All of the above? ‫דניאל‬

31. Choose the wrong answer about adolescence:


A. According to The Society for Adolescent Medicine, is defined as a period from 10 to 25 years of age.
B. Is characterized by 3 developmental stages: early, middle, late adolescence.

41. During regular checkup of an 8-year-old child, you note a loud first heart sound with a fixed and widely split
second heart sound at the upper left sternal border that does not changed with respirations. The patient is otherwise
active and healthy. Which of the following heart lesions most likely explains these findings?
A. Atrial septal defect (ASD)- ‫מאור‬
B. Ventricular septal defect (VSD)
C. Isolated tricuspid regurgitation
D. Tetralogy of Fallot
E. Mitral valve prolapse

44. Choose the correct answer about enterobiasis:


A. Humans are the only natural hosts
B. Is spread by the fecal-oral route via contaminated toys, clothes.
C Is caused by Enterobius vermicularis
D. Could be treated with Mebendazole or Albendazole
E. All of the above ‫מאור‬
13. An 11-year old girl presents with bloody diarrhea, vomiting and diffuse abdominal pain. Her examination is … for
…mal diffuse abdominal tenderness. Her white blood cells count is 12,000 her hemoglobin is 8 g/dl, and her platelets
count is 56,000, her creatinine is 1.8. all of the following is TRUE about the patients underlying disease EXCEPT:

15. Which of the following is the most likely cause of pneumonia in a gradually well- appearing 8-year-old with a
gradually progressive cough over two weeks and bilateral crackles on lung exam?
Mycoplasma

60. the most common concomitant malformations in down syndrome are:


A. defects of the CNS
B. defects of the skeletal system
C. heart defects- ‫שירן‬
D. kidney defects
E. vision defects

106. In a 3 months year old child in a general good health hypoglycemic seizures were observed. In differential
diagnosis which of the hormonal abnormalities should be considered:
Growth hormone deficiency

107. Hormones secreted by the hypothalamus are:


GnRH, GHRH, and CRH

108. The most common cause of acute stridor in children is:


Viral laryngotracheobronchitis (croup)

65. This is an x - ray of a 3 - year - old boy who presented with fever and abdominal pain of 2-3 days
. What is the abnormality you see on the chest x - ray ?
A. Left sided pleural effusion
B. Left basal consolidation
C. Collapse left lower lobe
D. Diaphragmatic hernia
E. Cardiomegaly and congestive heart failure

‫ שאלות שהמרצה שלחה‬40 ‫קובץ‬


21. The treatment recommended for acute sublaryngeal infection may include :
1 ) intramuscular dexamethasone ;
2 ) inhalled budesonide ;
3 ) inhalled adrenaline ;
4 ) oral prednisone ;
5 ) intramuscular adrenaline .
A. 1,3
B. 2,3
C. 2,4,5
D. all of the above
E. 1,2,3,4

22. A control chest radiography may be indicated in children treated for pneumonia in the case of :
1 ) persistent abnormalities on auscultation ( at the end of treatment ) ;
2 ) presence of lung abscess in previous exam ;
3 ) presence of atelectasis in previous exam ;
4 ) presence of pleural exudate in previous exam ;
5 ) Severe course of pneumonia .
A. 1,3
B. 2,3
C. 2,4,5
D. all of the above
E. 1,2,3,4

23. The parents checked at evening hours into emergency dept. with a 12-year old son who suffered from nausea,
vomiting, pain during micturition that radiated to the left groin area, and left lumbar pain since morning. On physical
examination, tenderness in the abdominal area and positive Goldflam syndrome on the left side were noted.
Laboratory test results revealed normal blood cell count and kidney function, increased CRP, and erythrocyturia in
urinalysis. Abdomen ultrasound showed left-sided hydronephrosis. The left ureter was not visible. The right diagnosis
is:
A. urinary system stones
B. acute pyelonephritis
C. rapidly progressive glomerulonephritis
D. peritonitis .
E. SARS - CoV - 2 infection

24. Indicate which of the statements regarding children with confirmed COVID 19 infection are true :
1 ) over 80 % of the children tested have mild infection ( are mildly symptomatic or asymptomatic ) ;
2 ) the high risk groups include , between others , newborns , children with chronic kidney disease , children with
obesity ;
3 ) infants and children under 2 years of age always need to be seen by a doctor ;
4 ) oxygen saturation measurement is an essential part of the examination ;
5 ) the markers that can be useful to monitor the cytokine storm include IL - 6 , ferritin , LDH , and D - dimers
A. all of the above
B. 1,2,3,4
C. 3,4,5
D. 2,3,4,5
E. 1,2,4

25. Initial ambulatory treatment for severe asthma exacerbation in children under 5 years of age consists in :
A. doubling the dose of inhaled steroids
B. administration of inhaled short - acting B - mimetics
C. administration of inhaled long - acting B - mimetics
D. addition of anti - leukotriene drugs in treatment
E. administration of oral long - acting theophylline
45. The first line of therapy for an acute asthma in children is:
A. inhaled anticholinergic medicine
B. inhaled short-acting beta2 agonist- ‫שירן‬
C. intravenous antihistamine medicine
D. oral antihistamine medicine
E. antuleukotriene medicine

26. A 10 - year - old child with diabetes diagnosed one year ago was completely vaccinated against hepatitis B in the
infancy. The parents ask whether the child needs the revaccination against hepatitis B due to the diabetes. Which is
the right answer?
A. the child does not need any further vaccinations against hepatitis
B. concentration of anti - HBs antibodies should be measured - if the result is higher than 10 IU / L the child does not
need any additional doses of HepB vaccine vaccine
C. concentration of anti - HBs antibodies should be measured - if the result is higher than 100 IU / L the child does not
need any additional doses of HepB
D. one booster dose of HepB vaccine should be given
E. the child should again receive the full 3 - dose schedule at 0 , 1 and 6 months

28. Indicate the false statement concerning chemoprophylaxis after the contact with a patient infected with Neisseria
meningitidis :
A. it is recommended for anyone who stayed in the patient's home or slept in the same room during 7 days before the
onset of the symptoms
B. vaccination should be considered in not vaccinated children
C. it consists in acyclovir given orally at a dose of 4 x 200 mg
D. it should be applied to close contact persons preferably within 24 hours , but not later than 2 weeks after the onset
of the symptoms
E. children should receive rifampicin or ceftriaxone ..

30. Indicate the true statement concerning newly diagnosed immune thrombocytopenia :
A. is an indication for an immediate start of immunosuppressive treatment
B. treatment of choice is splenectomy
C. it can be diagnosed only if the platelet count in the peripheral blood is <100x10³ uL and disorders are excluded that
might suggest other reason for thrombocytopenia (e.g. lymphadenopathy, hepatomegaly, anemia, abnormal WBC)
D. when the platelet count is < 50 x 10³ / uL there are indications platelet concentrate transfusion
E. in order to confirm the immune background it is necessary to perform tests for the presence of anti - platelet
antibodies in the blood

31. Which of the following signs and symptoms is not typical of iron - deficiency anemia ?
A. headaches and dizziness
B. weakness
C. cardiac systolic murmur
D. angular cheilitis
E. bradycardia

33. A 7 - month - old boy presented with a 2 - day history of fever up to 39 ° C . General condition was good . Physical
exam revealed pharyngitis only . On the day 3 temperature normalized and multiform macular , pink - red rush
fading under pressure was observed . What is the most probable etiology of the presented case ?
A. streptococci ( group A )
B. human herpes virus type 6 ( HHV - 6
C. varicella - zoster virus
D. Ebstein - Barr virus
E. measles virus

34. Fever and small itchy blisters that form crusts within 7 days are the symptoms of :
A. erythema infectiosum
B. impetigo
C. HSV infection
D. chickenpox
E. Measles

35. In acute pancreatitis , the indications for initiating antibiotic therapy include :
1 ) any case of acute pancreatitis related to choledocholithiasis ;
2 ) any case of alcohol - related acute pancreatitis ;
3 ) con ed presence of infected necrosis in the course of acute pancreatitis ;
4 ) pancreatic pseudocysts as a complication in acute pancreatitis .
A. 1,3,4
B. 2,3,4
C. 2,4
D. 3 only
E. 4 only

36. The first - choice drug used in treatment of acute liver failure related to paracetamol (acetaminophen) overdose is:
A. N - acetylcysteine
B. BAL
C. vitamin E ( high doses )
D. glutathione
E. barbiturate

37. A 3-year-old boy was admitted to emergency dept. due to high fever of >39.5°C lasting for two days, abdominal
pain, apathy, lack of appetite and urine odor. Urinalysis showed leukocyturia and the presence of nitrites. Indicate
appropriate treatment :
A. hospitalization and intravenous ampicillin
B. oral ciprofloxacin for 7 days
C. second- or third - generation cephalosporin , intravenously , orally or sequentially ‫ זה פיאלונפריטיס לא ציסטיטיס‬.‫דניאל‬
‫לכן נותנים צפלוספורנים דור שני\שלישי‬.
D. sulphamethoxazole with trimethoprim for 5 days
E. nitrofurantoin for 5 days

NEW BTs
1. The 5year old boy is brought to the emergency room with the symptoms of sudden onset of hemiparesis, headache,
speech disturbances and central VII nerves palsy at the side of hemiparesis. The family history and the boy’s history
is uneventful. A week before he presented the symptoms of upper respiratory tract infection. The most likely cause of
described symptoms is:
a. Ischemic stroke
b. Hemorrhagic stroke
c. Encephalitis ‫ דניאל‬, -‫ נראה לי זה‬RSV ‫יכול לגרום לאאנספליטיס איתי‬
d. Posterior fossa tumor

2. Transient ischemic attacks (TIA):


a. May last shorter than 15 minutes
b. Are caused by the risk factors similar to ischemic stroke
c. Their incidence in childhood population is unknown
d. All mentioned above

3. The picture given below represents the thrombosis of:


a. Sagittal superior sinus
b. Left sigmoid sinus
c. Right sigmoid and transverse sinus
d. Galen vein congenital malformation
4. When simple febrile seizure occur:
a. Give PR fenobarbital (3mg/kg)
b. Give antipyretic drug
c. Give PR diazepam (0.5mg/kg)
d. B and C are true ‫דניאל‬

5. The first line drug in typical absence epilepsy is:


a. Carbamazepine
b. Valproate
c. Topiramate
d. Etosuccimide - ARI

6. The first line drug in myoclonic epilepsy: 524 ‫בספר הפולני עמ‬
a. Lamotrygine
b. Oxacarbazepine
c. Clonazepam
d. Valproate - ARI

7. The side effects of phenytoin are following except one: can cause Hypothyroidism, Stevens-Johnson syndrome
a. Gingival hypertrophy Gingival hyperplasia occurs in 10–30% of patients treated with phenytoin.
b. Ataxia
c. Liver encephalopathy - ARI
d. Hirsutism Fetal hydantoin syndrome

8. The most useful in recognition of epilepsy: 522 ‫בספר הפולני עמ‬


a. MRI
b. CT
c. EEG - ARI
d. SPECT

9. The risk factors of cerebrovascular disorders in children are:


a. Congenital heart disorders
b. Acquired heart disorders
c. Malformations of brain vasculature
d. All mentioned above

10. The secondary prevention of ischemic stroke means the treatment with: 533 ‫בספר הפולני עמ‬
a. Acenocumarol
b. Aspirin - ARI- ‫ליאל מסכימה‬
c. Antibiotic
d. Hepari

11. Magnetic resonance imaging is useful in the evaluation of patients with prog--- encephalopathy. Some of
metabolic disorders in clinical picture has leukod….. which of the listed below diseases has leukodystrophy.
a. Lesch-Nyhan disease
b. Werding-Hoffman disease
c. Alpers disease
d. Adrenoleukodystrophy
e. Pompe diseas
12. Which of listed below disorders are caused by mitochondrial dysfunction
a. MERRF
b. Leigh disease
c. MELAS
d. Alpers disease
e. All above- ‫ ליאל‬-492 ‫בספר הפולני עמ‬

13. Evaluation of the very long chain fatty acids in serum is diagnostic to: 488 ‫בספר הפולני עמ‬
a. Peroxisomal diseases ‫דניאל‬
b. Lysosomal storage diseases
c. Mitochondrial disorders
d. Organic acid disturbances
e. Disturbances of fatty acids beta oxidation

14. One of the supportive therapies in inborn errors of metabolism is bone marrow transplantation. It can be performed
in asymptomatic patients with:
a. Phenylketonuria
b. XL-Adrenoleukodystrophy ‫דניאל‬
c. Maple syrup disease
d. Galactosemia
e. Pompe disease

15. A y-7year old boy was admitted to the hospital because of pneumonia. In examination you have noticed: calf
hypertrophy, hyperlordosis and wadd… laboratory test creatine kinase (CK) was markedly elevated. It is a highly
possible the boy has: 529 ‫בספר הפולני עמ‬
a. Werding-Hoffman disease
b. Myasthenia gravis
c. Gullian-Barre syndrome
d. Duchenne dystrophy- ‫ליאל‬
e. Poliomyelitis

16. The most effective treatment in Guillain-Barre syndrome: 527-528 ‫בספר הפולני עמ‬
a. Plasmapheresis
b. Antibiotic therapy
c. Antiviral therapy
d. Glucocorticoids therapy
e. Antiepileptic therapy

40. The Guillain-Barre syndrome: 527 ‫בספר הפולני עמ‬


a. This is acute inflammatory polyradiculoneuropathy- ‫נכון‬
b. Is the most common cause of floppy paresis in developed countries.
c. The treatment of choice is intravenous immunoglobulin administration- ‫נכון‬
d. An alternative treatment method is plasmapheresis- ‫נכון‬
e. All the answers are correct- ‫ליאל‬

17. Which of listed below symptoms is most characteristic in spinal muscular atrophy: 527 ‫בספר הפולני עמ‬
a. Elevated creatine kinase in serum
b. Proximal muscle wasting and weakness ‫דניאל‬
c. Cardiomyopathy
d. Myopathic pattern of EMG abnormalities
e. Eyelid ptosis.
18. Werding-Hoffman disease is: 52 ‫בספר הפולני עמ‬
a. Caused by mutations in SMN1 gene
B. Caused by mutations in DMD gene
C. Caused by mutations in NF1 gene
D. Triggered by a preceding viral or bacterial infection
E. We don’t know the pathogenesis of this disorder

19. Duchenne’s muscular dystrophy is caused by mutations in the DMD gene. The.. of this abnormal structure or lack
of dystrophin. What is the major role of ..?
A. Stabilization of extracellular matrix
B. Stabilization of sarcolemma
C. Activation of post transcriptional processing of mRNA
D. Integration of neuromuscular junction
E. Stimulation of cellular axonal transport

20. Clinical manifestations of the classic forms of Sturge Weber syndrome include: 536 ‫בספר הפולני עמ‬
a. Flat face hemangioma
b. Hemangioma of the soft meninges of the brain
c. Hemispheres or focal epileptic seizures
d. Correct answer A+B
e. Correct answer A+B+C ‫דניאל‬

21. The extrahepatic causes of infant cholestasis do not include:


a. Biliary atresia
b. Cholelithiasis
c. Alagille syndrome ‫ דניאל‬Intrahepatic biliary duct aplasia/hypoplasia
d. Spontaneous perforation of the common bile duct
e. Common bile duct cyst

23. Symptoms of autoimmune hepatitis include


a. Jaundice and / or hepatosplenomegaly- ‫נכון‬
b. Weakness, lack of appetite, weight loss
c. Bone and joint pain
d. Coagulation disorders – prothrombin system disorders
e. All symptoms of the above ‫ דניאל‬-‫ליאל מסכימה‬

24. Portal hypertension may develop as a consequence of: https://www.ncbi.nlm.nih.gov/pmc/articles/PMC5436736/


a. Pre-hepatic block
b. Intrahepatic block
c. Suprahepatic block
d. All the answers are correct- ‫ליאל‬
e. None of the answers are correct

28. Which of the following causes of gastroenteritis have been associated with seizures?
a. Salmonella
b. Shigella
c. Campylobacter
d. Adenovirus- ‫ דניאל‬-‫ליאל נוטה להסכים‬
e. Klebsiella pneumoniae
29. An 11-year old girl presents with bloody diarrhea, vomiting, and diffuse abdominal pain. Her examination is notable
for minimal, diffuse abdominal tenderness. Her white blood cell count is 12000, her hemoglobin is 8g/dl, and her
platelet count is 56000. Her BUN is 44 and her creatinine is 1.8. all of the following is TRUE about this patient’s
underlying disease EXCEPT:
a. Associated with bacterial infection
b. Most common cause of acute renal failure requiring dialysis
c. Frequently accompanied by neurologic symptoms
d. Steroids helpful if given early in the course of illness ‫דניאל‬
e. The similarities between HUS, aHUS, and TTP make differential diagnosis essential

30. Which of the following are clinical features of Patau syndrome? ‫)י‬128( 141 ‫בספר הפולני עמ‬
a. Low birthweight, small mouth and chin, short sternum, cardiac and renal malformations-
Edwards syndrome
b. Structural defect of brain, cleft lip and palate, polydactyly, cardiac and renal malformations-
‫ דניאל‬-‫ליאל מסכימה‬
c. Short stature, neck webbing or thick neck, widely spaced nipples, renal anomalies
d. Short neck, hypotonia, epicanthic folds, small earsm flat occiput, third fontanella
e. Macrocephaly, macroorchidism, long face, prominent mandible

31. Kawasaki disease (KD) is an acute febrile illness of unknown etiology that primarily affects children younger then 5
years of age. Patient with KD has a fever lasting at least 5 days and at least 4 of the following clinical signs:
a. Rash, cervical lymphadenopathy (at leats 1.5 cm in diameter), bilateral conjunctival injection, oral mucosal changes,
peripheral extremity changes. ‫ ליאל‬-)278( 291 ‫בספר הפולני עמ‬
b. Rash diarrhea, vomiting, bilateral conjunctival injection, peripheral extremity change
c. Headache, “strawberry tongue”, rash, bilateral conjunctival injection, oral mucosal changes.
d. Cervical lymphadenopathy (less then 0.5cm in diameter), rash, diarrhea, bilateral conjunctival injection, oral
mucosal changes
e. Arthritis, rash, headache, cervical lympahdnopathy (at leats 1.5cm in diameter), bilateral conjunctival injection.

32. Erythema infectosum (EI) is also known as Fifth disease. Choose the correct answer: 286 ‫בספר הפולני עמ‬
a. Is caused by parvovirus B19
b. Transmission is via respiratory droplets with attack rates among close contacts up to 50%
c. Incubation is 4-14 days
d. The exanthem is a characteristic “slapped cheeks” appearance
e. All answers is correct- ‫ליאל‬

35. Choose the correct answer: ‫מהמצגת‬


a. Acetaminophen is antipyretic drug which is forbidden in infants younger < 3 months old
b. Ibuprofen might be administrated intravenously
C. ibuprofen might be used in newborn when treatin fever
D. the doses of acetaminophen are: 10-15 mg/kg every 4-6 hours- ‫ליאל ודניאל‬
E. metamizole is a very safe antipyretic drug broadly used in pediatric departments in dosage 1-2g every 3 hours in
infant with weight<10ibs
37. Uveitis is most common in: ‫)י‬282( 295 ‫במצגת ובספר הפולני עמ‬
a. Juvenile systemic lupus erythematosus
b. Rheumatic fever
c. Kawasaki’s disease
d. Juvenile idiopathic arthritis- ‫ליאל‬
e. Sjogren’s disease
38. The diagnostic criteria for systemic lupus erythematosus are not: ‫במצגת‬
a. Butterfly-shaped erythema
b. Disc erythema
c. Migratory erythema- ‫ליאל‬
d. Photosensitivity
e. Serosa inflammation

39. A widespread maculopapular rash from days 2 to 3 of life that disappear spontaneously around the first week are:
a. Neonatal erythema ‫דניאל‬
b. Neonatal acne
c. Milia- in the first few weeks of life
d. Ebstain pearls
e. “prickly heat”

41. A 14-year old girl was admitted to the department of pediatrics because of anemia. Weakness and weight
deficiency. A history of chronic diarrhea (negates blood admixtures in the stool). Laboratory test showed anemia,
moderately elevated inflammatory markers (CRP) , and high fecal calprotectin. A physical a bscess of the perianal
area was visualized. The whole clinical picture first suggests: ‫)י‬253( 266 ‫בספר הפולני עמ‬
a. Specific enteritis of bacterial etiology
b. Celiac disease
c. Ulcerative colitis- ‫ ירידה במשקל יותר נפוצה בקרוהן‬.‫בדרכלל דם בצואה‬
d. Crohn’s disease- ‫ליאל‬
e. SIBO (bacterial hypertrophy of the small intestine)

42. Generalized lymphadenopathy is not caused by: 285-286 ,167 ‫בספר הפולני עמ‬
a. CMV
b. EBV
c. Hepatitis B virus- ‫ליאל‬
d. Toxoplasma Gondii
e. Varicella – zoster virus

43. Which pathogens are most common cause the pneumoniae in the 3 months to 5 years age group?
a. M.Pneumoniae, S.pneumoniae, Chlamydophila pneumoniae.
b. RSV, parainfluenza viruses, adenoviruses, S.pneumoniae- ‫ ליאל‬-)305( 2018 ‫במצגת ובספר הפולני עמ‬
c. Influenza viruses, M.pneumoniae
d. Group B streptococcus, E.coli, S.pneumoniae
e. E.coli, Listeria monocytogenes

44. Eating disorders with two-phase course (initially no sucking reflex, feeding difficulties, and after 2 years of age
increased appetite with a rapid development of obesity) are characteristics of:
a. Congenital hypothyroidism
b. Spinal muscular atrophy
c. Prader-Willi syndrome- ‫ליאל‬
d. Cerebral palsy
e. Down syndrome

45. A 5-Year old boy presents with abdominal pain. Potential cause of his abdominal pain include:
a. Pneumonia
b. Strep throat
c. Testicular torsion
d. Diabetic ketoacidosis
e. All of the above- ‫ ליאל‬-)236( 249 ‫ספר פולני עמ‬

48. The least probable symptoms of cystic fibrosis is:


a. Rectal prolapse
b. Chronic sinusitis
c. Malabsorption syndrome
d. Intussusception- ‫ליאל‬
e. Nasal polyps occurrence

49. Which of the following heart defects is a cyanotic one?


a. Ventricular septal defect with left-right shunt
b. Transposition of great arteries- (TGA) -‫ליאל‬
c. Atrial septal defect type III (ASD III)
d. Atrial septal defect type I (ASD I)
e. Patent ductus arteriosus with left right shunt.

51. Rapid, shallow breaths, reduced chest movement on affected side, dull percussion and crackles are chest signs
observed in:
a. Asthma
b. Croup
c. Bronchiolitis
d. Pneumonia- ‫ליאל‬
e. None of the above

53. What is the age limit when child can feed self/using spoon?
a. 2 months
b. 4 months
c. 10 months
d. 18 months- ‫ליאל‬
e. 24 months

54. Which neurological features of cerebral palsy is the most common? ‫)י‬58( 71 ‫בספר הפולני עמ‬
a. Spastic (bilateral, unilateral)- ‫ליאל‬
b. Dyskinetic
c. Ataxic
d. B+C
e. Mixed pattern
46 The MOST common type of cerebral palsy is
A. ataxis 4%
B. hypotonic
C. mixed 10%
D. spastic 80%‫מאור‬
E. A and D correct
E. Corticosteroid systemic

55. Which of the following is/are true?


a. There is good evidence from randomized controlled trials that mist therapy (epinephrine) is not effective for the
treatment of croup.
b. Antibiotics are indicated in the treatment of croup. Most common pathogen: parainfluenza viruses (75% of cases)
C. Nebulized albuterol is effective in the treatment of croup.
D. Dexamethasone has been shown to be effective in the treatment of croup. ‫מאור‬
e. Every patient with a croup requires treatment at the hospital at the intensive care unit.

58. A 7-year old boy: diabetes mellitus type 1 is diagnosed. What would the most appropriate initial management be if
serum blood sugar is 700, serum and urine ketones are large, pH is 7.12 and K is 4.5?
a. Bolus of IV insulin
b. Bolus of IV bicarbonate because of pH
c. Bolus of normal saline plus K ‫ דניאל‬Fakher
d. A subcutaneous dose of insulin

59. At what age in a Caucasian girl would you be concerned about precocious puberty?
a. 10,5 y
b. 11 y
c. 9.5 y
d. 6 y- ‫ ליאל מסכימה‬-‫דניאל‬
e. In Caucasian girls there is no age limit
61. Tonsillectomy should be considered in a child with: ‫דניאל‬-‫ב נכונים‬+‫ א‬-‫ ואינטרנט‬301 ‫לפי ספר קורס עמ‬
a. Recurrent tonsilitis
b. Obstructive sleep apnea
c. Rheumatic fever
d. B+C correct
e. All of the above

63. the proper treatment in acute poststreptococcal glomerulonephritis is: ‫)י‬362( 375 ‫ספר פולני עמ‬
a. 10 days of trimethoprim-sulfamethoxazole
b. Corticosteroids high dose until proteinuria resolves (about 2 ..)
c. NSAIDS for 2-4 weeks
d. There is no specific treatment ‫ דניאל‬-‫ ליאל נוטה להסכים‬,Fakher

64. The most common fungal infection in humans is:


a. Coccidiomycosis,
b. Histoplasmosis
c. Aspergillosis
d. Candidiasis ‫ דניאל‬-‫ ליאל‬-‫נוטה להסכים‬
e. Scabies

66. All of the following statements about sinus arrhythmia are true except:
a. It is normal in healthy children
b. It is acceleration during respiration, slowing on inspiration ‫דניאל‬.
c. Heart rate changes up to 30 beats/min
d. It is detectable as a cyclical change in heart rate with respiration
e. It can be detected on auscultation and ECG

67. Most infants lose weight immediately after birth. Normal term infants generally regain their birth weight by:
a. 24 hours
b. 48 hours
c. 72 hours
d. 3-5 days
e. 7-10 days- ‫ דניאל‬-‫ליאל מסכימה‬
68. The most common congenital heart disease is:
a. Aortic stenosis- 5%
b. Pulmonary stenosis- 7%
c. Atrial septal defect- 7%
d. Tetralogy of Fallot- 5%
e. Ventricular septal defect- ‫דניאל‬

69. All of the following statements about puberty are true except:
a. Breast development is the first sign in females
b. Penile growth is the first sign in males ‫ דניאל‬-‫ליאל מסכימה‬
c. A testicular volume of 2ml is prepubertal
d. Some breast development may occur in normal pubertal males
e. A testicular volume of 4ml is pubertal

71. What is the normal respiratory rate in infants?


a. 30-50
b. 25-40
c. 20-25
d. 12-20
e. >50

78. You are performing infant CPR. What is the ratio of chest compressions to ventilations?
a. 30 chest compressions to 2 ventilations ‫ לבד דניאל‬-‫ ליאל‬-”‫לדעתי לזה התכוונו בשאלה כשכתבו “אתה מבצע החייאה‬
b.15 chest compressions to 2 ventilations ‫ בצוות דניאל‬.‫ פאכר‬,‫ הייתה מדגישה את זה‬,‫אם רצתה איש צוות אחד‬
c. 30 ventilations to 2 chest compressions
d. 15 ventilations to 2 chest compressions
e. Only 30 chest compressions

You might also like